The Flat Earth Society

Flat Earth Discussion Boards => Flat Earth Investigations => Topic started by: Curiosity File on September 29, 2018, 07:15:30 AM

Title: Earths Curvature and spin effect on long range ballistics.
Post by: Curiosity File on September 29, 2018, 07:15:30 AM
The Coriolis effect on ballistics is yet another thing that proves the Earth spins and has curvature.
At 1,000 yards the target raises or drops as much as 6"depending on direction, east or west, coming toward you it raises and away from you it drops.
Shooting north or south the target does not drop or raise, rather moves left or right 3" depending on direction.
In addition at the poles off set is greater.
This would not be possible on a stationary flat surface. 

https://www.straightdope.com/columns/read/2944/how-does-the-earths-rotation-affect-the-path-of-a-bullet/

Title: Re: Earths Curvature and spin effect on long range ballistics.
Post by: Curiosity File on September 29, 2018, 09:34:09 AM
This is even more fascinating. The fact that the velocity is greater at the equator than it is as as you get closer to the poles is also proof of curvature/a globe. 1,037 mph at the equator vs, "For example, at 45 degrees North (halfway between the Equator and the North Pole, a.k.a. along Montana’s southern border) you’re only moving about 733 mph."
So when you fire a bullet in the northern hemisphere it deflect to the right of the target no matter if you are facing north or south. This is due to the differences in velocity you and the gun are traveling when the bullet leaves the barrel.
If when you fire to the north from the equator it's moving to the east(to the right) at a faster speed than the location of the target, thus deflects to the right of the target because the target hasn't caught up to it.
If you shot towards the south,(towards the Equator) the eastward movement of the bullet is slower than that of the target closer to the equator which causes the bullet to hit to the right.
This is reversed in the southern hemisphere.

https://loadoutroom.com/thearmsguide/external-ballistics-the-coriolis-effect-6-theory-section/

https://www.thetruthaboutguns.com/2018/01/jeremy-s/coriolis-effect-for-beginners-extreme-long-range-shooting-for-beginners/

Irrefutable proof the Earth is spherical and spins at a high rate of speed.
 
   
Title: Re: Earths Curvature and spin effect on long range ballistics.
Post by: sandokhan on September 30, 2018, 03:26:42 PM
Curvature effects = DePalma spinning effect on ballistics.

https://www.theflatearthsociety.org/forum/index.php?topic=30499.msg2029817#msg2029817 (part I)

https://www.theflatearthsociety.org/forum/index.php?topic=30499.msg2032069#msg2032069 (part II, formula)
Title: Re: Earths Curvature and spin effect on long range ballistics.
Post by: Curiosity File on September 30, 2018, 10:44:06 PM
Curvature effects = DePalma spinning effect on ballistics.

https://www.theflatearthsociety.org/forum/index.php?topic=30499.msg2029817#msg2029817 (part I)

https://www.theflatearthsociety.org/forum/index.php?topic=30499.msg2032069#msg2032069 (part II, formula)

Hi sndokhon. I appropriate your response. However that's a lot to read through and maybe even hard to grasp.
Maybe you could help me out here, as it was hard enough for me to understand why you hit to the right of the target as much as 3" shooting from the equator to the north at 1,000 yards.  Then again hitting 3" to the right of the target shooting from north of the equator south to the equator.
Maybe you could be gracious enough to pull something out of all that info you shared that actually disproves that happens and that the trigonometry used to calculate this is wrong?
Or maybe something that proves how this can happen on a flat Earth that's stationary?
Take one step at a time.

Thank you in advance.     
Title: Re: Earths Curvature and spin effect on long range ballistics.
Post by: sandokhan on October 01, 2018, 05:53:04 AM
The Coriolis effect on ballistics is yet another thing that proves the Earth spins.

The Coriolis force exists only when one uses a rotating reference frame.

Either the Earth's supposed rotation, OR the ether drift's rotation above the surface of the Earth.

"Artillery projectiles are spinning at a very high speed when they exit the barrel of the gun. The spinning stabilises the projectile in flight and makes it more accurate."

http://nigelef.tripod.com/fc_ballistics.htm

For artillery projectiles spin rates in the order of 20,000 revolutions per minute are needed, rifle bullets are an order of magnitude greater.

The rotation of the projectile (its spinning rate) will radically alter both its mass and its inertia.

The rotation produces a TORSION FIELD which will attract the Whittaker potential waves (ether longitudinal waves) thus forming an ether vortex around the projectile which will impart antigravitational properties.

The magnitude of this effect is totally unaccounted for by modern science, in fact it is attributed to curvature calculations.

The high spinning rate of the projectile will be subjected to the Coriolis effect of the ether drift.

 Only a light interferometer, which registers both the Coriolis effect and the rotational Sagnac effect, can answer the question: does the Earth rotate around its own axis, or does the Sun orbit above the flat surface of the Earth?

The formula for the Coriolis effect applied to light interferometers:

https://www.theflatearthsociety.org/forum/index.php?topic=30499.msg2068289#msg2068289

https://www.theflatearthsociety.org/forum/index.php?topic=30499.msg2069660#msg2069660

The formula for the Sagnac effect applied to light interferometers which are located away from the center of rotation:

https://www.theflatearthsociety.org/forum/index.php?topic=30499.msg2070082#msg2070082

https://www.theflatearthsociety.org/forum/index.php?topic=30499.msg2070907#msg2070907

Title: Re: Earths Curvature and spin effect on long range ballistics.
Post by: sandokhan on October 01, 2018, 01:06:35 PM
It should be noted that without the use ether drift theory, one cannot explain the formula for the long-range ballistic formula on a flat earth:

(http://image.ibb.co/fre6Qx/art1.jpg)
(http://image.ibb.co/et6syH/art2.jpg)
(http://image.ibb.co/hBYGQx/art3.jpg)

No other FE has ever dared to tackle this important issue, long-range ballistics on a flat earth, because the difference between the two formulas (RE vs FE) is very important.

With the help of the ether drift theory, we can MODIFY the g acceleration and replaced it by f(k):

R = {vo2(sin 2θo)}/f(k)

k is the variable electrogravitational value, which depends on the altitude, the atmospheric ether tide, the density of ether at a certain altitude, and the spin rate

The curvature factor is ~EQUAL to the antigravitational effect produced by the spin rate of the projectile which forms a torsion field which partially cancels out the g force.
Title: Re: Earths Curvature and spin effect on long range ballistics.
Post by: Humble B on October 01, 2018, 04:12:02 PM
Dear mister Sandokhan,

Am I right to say that your theory claims that there is a relation between the spin of a projectile around its own axis and a Coriolis force changing the trajectory of that projectile:



For artillery projectiles spin rates in the order of 20,000 revolutions per minute are needed, rifle bullets are an order of magnitude greater.

The rotation of the projectile (its spinning rate) will radically alter both its mass and its inertia.

The rotation produces a TORSION FIELD which will attract the Whittaker potential waves (ether longitudinal waves) thus forming an ether vortex around the projectile which will impart antigravitational properties.

The magnitude of this effect is totally unaccounted for by modern science, in fact it is attributed to curvature calculations.

The high spinning rate of the projectile will be subjected to the Coriolis effect of the ether drift.

In that case I've a few question I like you to answer:

1- Why formula's to calculate the Coriolis force in ballistics do not take the spin of the projectile in account, only its mass, velocity, latitude and hemisphere? (and of course the angular velocity of the spinning surface, which is considered a constant of 0.0000727 1/s) https://www.omnicalculator.com/physics/coriolis-effect (https://www.omnicalculator.com/physics/coriolis-effect)

2- Why the Coriolis effect on earth is not only observed on spinning projectiles, but similarly also non-spinning projectiles?

3- The spin of ballistic missiles is desired because the gyroscopic effect of that spin gives that projectile more stability and a more precise trajectory. But if that spin is "forming an aether vortex around the projectile" making it "sensitive for the Coriolis effect", then the opposite is true; the spin is responsible for pushing the projectile out of his initial trajectory. Then why our military is using spinning bullets when they are less accurate because of the Coriolis effect?

4-
The Coriolis force exists only when one uses a rotating reference frame.
Either the Earth's supposed rotation, OR the ether drift's rotation above the surface of the Earth.
The high spinning rate of the projectile will be subjected to the Coriolis effect of the ether drift.

If I understand this well, your theory is explaining that a rotating aether above a stationary earth is what creates the Coriolis force. But how does that explain that on the northern hemisphere a bullet traveling from south to north is drifted to the east, while that same bullet would be drifted to the west when it moves from north to south?
And how does this spinning aether explain that on the southern hemisphere this effect is opposite: a bullet traveling from south to north is drifted to the west, while that same bullet would be drifted to the east when it moves from north to south ->

(https://docs.google.com/drawings/d/e/2PACX-1vQl1o3N9s8aDIvsebDaMVjH0MZS6qQAxXOgc2PVtFAXA_yojtKSQSVLS5ljqkrYQYerO2EsGgyufl4S/pub?w=452&h=700)

If a spinning aether would create a Coriolis force it would be more logical that this force is always working in the same direction as the spin of the eather, and the resulting Coriolis effect would always be in the same direction, no matter if you are on the northern or southern hemisphere, or firing to in north or south direction ->

(https://docs.google.com/drawings/d/e/2PACX-1vSuocomzZMmVTQ3LTbUlGjvF9fgKr2AeVcss5B_DnKDZ8z46-TGtiQX4VdYwOASPUt-IFmPvkc8j2b2/pub?w=452&h=700)

.
Title: Re: Earths Curvature and spin effect on long range ballistics.
Post by: sandokhan on October 01, 2018, 05:30:44 PM
Why formula's to calculate the Coriolis force in ballistics do not take the spin of the projectile in account, only its mass, velocity, latitude and hemisphere?

Because that is the classic formula for the Coriolis effect:

https://books.google.ro/books?id=xVXDjDVuhN4C&pg=PA94&lpg=PA94&dq=coriolis+effect+on+spinning+projectiles+vs+nonspinning+projectiles&source=bl&ots=BMHajnF2EG&sig=boUJ82BQYl8QTF41PAI7gTA97t8&hl=en&sa=X&ved=2ahUKEwiJpNKq1uXdAhWL1ywKHdbcAskQ6AEwC3oECAMQAQ#v=onepage&q=coriolis%20effect%20on%20spinning%20projectiles%20vs%20nonspinning%20projectiles&f=false

It simply registers the ether drift's effect on objects.

As for a careful investigation of the spinning rate in relation to the Coriolis force, this is a subject which has only recently begun to be researched:

http://www.appliedballisticsllc.com/Articles/ABDOC108_GyroscopicAndCoriolis.pdf

Why the Coriolis effect on earth is not only observed on spinning projectiles, but similarly also non-spinning projectiles?

Each and every object on Earth is subject to the effect of the Coriolis force of the ether drift. As I said, a careful study of this effect on spinning projectiles vs non-spinning projectiles is in its infancy.

The spin of ballistic missiles is desired because the gyroscopic effect of that spin gives that projectile more stability and a more precise trajectory. But if that spin is "forming an aether vortex around the projectile" making it "sensitive for the Coriolis effect", then the opposite is true; the spin is responsible for pushing the projectile out of his initial trajectory.

No.

The DePalma effect (gyroscopic effect) is the source of the stability and precision of the trajectory. However, given the fact that both the velocities and the spinning rates only slightly alter the gravitational acceleration factor/figure, this means that this antigravitational effect is equal to the earth curvature ballistics calculations.

The influence of the Coriolis force on spinning projectiles vs. non-spinning projectiles is a subject which has yet to be addressed in full force by modern science.

But how does that explain that on the northern hemisphere a bullet traveling from south to north is drifted to the east, while that same bullet would be drifted to the west when it moves from north to south?
And how does this spinning aether explain that on the southern hemisphere this effect is opposite: a bullet traveling from south to north is drifted to the west, while that same bullet would be drifted to the east when it moves from north to south ->


Ether drift: a center and two poles. From each pole to the center, we have two longitudinal waves, travelling in double torsion fashion, one has a left-handed spin, the other one a right-handed spin.

Therefore, the Coriolis effect (and the Eotvos effect) of the ether drift are based on these complex trajectories of the longitudinal scalar waves.

You have your information wrong: the deflection is not east or west, but specifically to the right or left with reference to the shooting direction. It doesn’t matter in which direction you shoot; it is a function of latitude and average bullet speed.

Now, the ether drift is latitude dependent.

So, if you launch a projectile, due north (south to north, in the northern semiplane), its path will be deflected slightly to the east, because of the difference in the velocity of the ether drift (different latitudes); if you launch the same projectile due south (north to south, in the northern semiplane), the path will be deflected slightly to the right of its true path, since now you again have to deal with the different velocities (different latitudes) of the ether drift.


Now, you are going to have to explain this.

Only a light interferometer, which registers both the Coriolis effect and the rotational Sagnac effect, can answer the question: does the Earth rotate around its own axis, or does the Sun orbit above the flat surface of the Earth?

The formula for the Coriolis effect applied to light interferometers:

https://www.theflatearthsociety.org/forum/index.php?topic=30499.msg2068289#msg2068289

https://www.theflatearthsociety.org/forum/index.php?topic=30499.msg2069660#msg2069660

The formula for the Sagnac effect applied to light interferometers which are located away from the center of rotation:

https://www.theflatearthsociety.org/forum/index.php?topic=30499.msg2070082#msg2070082

https://www.theflatearthsociety.org/forum/index.php?topic=30499.msg2070907#msg2070907

Why did Michelson make use of the Coriolis effect formula, while claiming all the while he was measuring the rotational Sagnac effect?

The Coriolis effect is a physical effect on the paths of the light beams; the Sagnac effect is an electromagnetic effect on the speeds of the light beams. Michelson recorded ONLY the physical effect, the Coriolis effect.
Title: Re: Earths Curvature and spin effect on long range ballistics.
Post by: Curiosity File on October 01, 2018, 06:09:01 PM
Thank you Humble B and  sandokhon for you replies.
Very interesting info. Call me intrigued.
Fascinating to say the least.

The model charts don't address, on the RE, why the change from equator to north and from north to equator always deflect to the right?
From my understanding this can only be explained , and calculated, by curvature?
As the over all circumference decreases from the equator towards the poles. Thus the spin rate decreases in direct relation to the decrease in circumference.
The sideways drift speed changes from launch location to target location.

Next we can look at the fact shooting west to east or east to west there is no sideways drift of the neither launch site, projectile or target location.
There is on raise or drop of the target depending on which direction you shoot.
Shooting east the target drops so you hit high.
Shooting west the target raises so you hit low.
How does the FET and models explain this?
 

     
Title: Re: Earths Curvature and spin effect on long range ballistics.
Post by: sandokhan on October 01, 2018, 06:17:53 PM
Next we can look at the fact shooting west to east or east to west there is no sideways drift of the neither launch site, projectile or target location.
There is on raise or drop of the target depending on which direction you shoot.
Shooting east the target drops so you hit high.
Shooting west the target raises so you hit low.
How does the FET and models explain this?


Yes, this is the Eotvos effect.

This is not the centrifugal RE force at all.

It has everything to do with the direction of both the laevorotatory scalar telluric longitudinal waves (antigravitational) and the dextrorotatory waves (gravitational).

Few scientists understand that the Eotvos effect cannot be explained by attractive gravitation.

The gravitational anomalies discovered by Roland Eotvos remain completely unexplained by modern science:

http://mek.oszk.hu/02000/02054/html/onehund.html
 
Title: Re: Earths Curvature and spin effect on long range ballistics.
Post by: Curiosity File on October 01, 2018, 06:25:04 PM
Next we can look at the fact shooting west to east or east to west there is no sideways drift of the neither launch site, projectile or target location.
There is on raise or drop of the target depending on which direction you shoot.
Shooting east the target drops so you hit high.
Shooting west the target raises so you hit low.
How does the FET and models explain this?


Yes, this is the Eotvos effect.

This is not the centrifugal RE force at all.

It has everything to do with the direction of both the laevorotatory scalar telluric longitudinal waves (antigravitational) and the dextrorotatory waves (gravitational).

Few scientists understand that the Eotvos effect cannot be explained by attractive gravitation.

The gravitational anomalies discovered by Roland Eotvos remain completely unexplained by modern science:

http://mek.oszk.hu/02000/02054/html/onehund.html

Intriguing. It will take me a while to sift through this. Thanks again
Title: Re: Earths Curvature and spin effect on long range ballistics.
Post by: Curiosity File on October 01, 2018, 06:58:58 PM
Simple calculations and real world affect is opposite to FET calculations.
We know exactly what happens in the real world when you fire a bullet to the east and west.
When you shoot to the east the target drops so you hit high.
On a FE model, with or without drift, the bullet ALWAYS hits low from gravity pull.
Likewise shoot to the west the bullet ALWAYS hits low.
If there were no drift the bullet would hit equally low shooting either direction.
With drift the bullet would hit lower shooting to the east because the target is moving away giving more time for the bullet to drop from gravity pull. This would be reversed shooting to the west.
Mathematical calculations match real world effect on a RE with rotation,(not drift), but is in error on a FE model with or without drift.
 
Title: Re: Earths Curvature and spin effect on long range ballistics.
Post by: Curiosity File on October 01, 2018, 09:04:50 PM
Next we can look at the fact shooting west to east or east to west there is no sideways drift of the neither launch site, projectile or target location.
There is on raise or drop of the target depending on which direction you shoot.
Shooting east the target drops so you hit high.
Shooting west the target raises so you hit low.
How does the FET and models explain this?


Yes, this is the Eotvos effect.

This is not the centrifugal RE force at all.

It has everything to do with the direction of both the laevorotatory scalar telluric longitudinal waves (antigravitational) and the dextrorotatory waves (gravitational).

Few scientists understand that the Eotvos effect cannot be explained by attractive gravitation.

The gravitational anomalies discovered by Roland Eotvos remain completely unexplained by modern science:

http://mek.oszk.hu/02000/02054/html/onehund.html

First i'd like to say how do you expect me to understand these experiments when modern scientists cann't. I also seriously doubt you understand it either as it appears to have nothing to do with ballistics and Coriolis.

Furthermore it is just and isea that a fifth force exists, it's hypothetical, theoretical, never been proven and came about 3 or 4 hundred years ago.

Interest in it picked up again in 1980. But no articles wrote about any findings or if they even took up experiments again..   

Also no summery of the information and no conclusive evidence was ever found.

Even so the hypothesis of this fifth force shows so such little effect I don't see how it would effect ballistics.

Also it still would be a constant and would not explain how the trajectory changes as it does in the real world depending on location and direction in which a projectile is launched.

So my summery is a 400 year old hypothetical theory of some mysterious force is weak to use to disprove a physically and mathematically proven fact.
 

 
   
Title: Re: Earths Curvature and spin effect on long range ballistics.
Post by: Curiosity File on October 01, 2018, 09:25:48 PM
I'd also like to point out that this 5th force, if it exists, is so weak that they were conducting extremely elaborate experiments that took long periods of time to get any results.
In comparison the force of gravity is so strong you can watch stuff fall to the earth while it has limited effect on seeding bullets.
So how do we expect such a weak force as the "EOTVOS" effect influence something as powerful a bullet?
Title: Re: Earths Curvature and spin effect on long range ballistics.
Post by: sandokhan on October 02, 2018, 08:31:24 AM
The gravitational anomaly discovered by Roland Eotvos is not the same as the Eotvos effect (vertical displacement of the projectile), yet they are caused by the same phenomenon.

E.T. Whittaker proved the existence of the longitudinal waves:

https://www.theflatearthsociety.org/forum/index.php?topic=30499.msg1994059#msg1994059

In comparison the force of gravity is so strong you can watch stuff fall to the earth while it has limited effect on seeding bullets.

Here is a force which is stronger than the force of gravity: the Biefeld-Brown antigravitational effect.

https://www.theflatearthsociety.org/forum/index.php?topic=30499.msg2031282#msg2031282

Or the Allais antigravitational effect:

https://www.theflatearthsociety.org/forum/index.php?topic=30499.msg760382#msg760382

Once that "fifth force" is activated (a very strong electrical field, sound, double torsion) it becomes much stronger than the force of terrestrial gravity.


"Well gentlemen, we will undertake this, although my conviction is strong that we shall prove only that the earth rotates on its axis, a conclusion which I think we may be said to be sure of already."

A. Michelson

This is the formula published by Michelson in 1904 and 1925:

(http://www.conspiracyoflight.com/Michelson-Gale_webapp/image004.png)

However, this is the CORIOLIS EFFECT FORMULA for circuital light beams.

Here it is:

Δt = 4AΩsinΦ/c^2 (where Φ is the latitude)

https://www.ias.ac.in/article/fulltext/pram/087/05/0071

The same formula was derived by Dr. Ludwik Silberstein in 1921:

https://www.theflatearthsociety.org/forum/index.php?topic=30499.msg2068289#msg2068289

That is, the CORIOLIS EFFECT upon the light beams is totally related to the closed contour area.

It is a physical effect, it deflects the path of the light beams.

By constrast, the rotational Sagnac effect is an electromagnetic effect, it modifies the speed of the light beams.

Michelson and Gale ONLY recorded the Coriolis effect of the ether drift upon the light beams; no rotational Sagnac effect was detected.

Here is the derivation of the rotational Sagnac effect:

https://www.theflatearthsociety.org/forum/index.php?topic=30499.msg2070082#msg2070082

https://www.theflatearthsociety.org/forum/index.php?topic=30499.msg2070907#msg2070907

The Sagnac effect formula is proportional to the radius of the Earth, while the Coriolis effect is proportional to the area of the interferometer and thus thousands of times smaller in magnitude than the Sagnac effect.
Title: Re: Earths Curvature and spin effect on long range ballistics.
Post by: Curiosity File on October 02, 2018, 05:15:24 PM
The gravitational anomaly discovered by Roland Eotvos is not the same as the Eotvos effect (vertical displacement of the projectile), yet they are caused by the same phenomenon.

E.T. Whittaker proved the existence of the longitudinal waves:

https://www.theflatearthsociety.org/forum/index.php?topic=30499.msg1994059#msg1994059

In comparison the force of gravity is so strong you can watch stuff fall to the earth while it has limited effect on seeding bullets.

Here is a force which is stronger than the force of gravity: the Biefeld-Brown antigravitational effect.

https://www.theflatearthsociety.org/forum/index.php?topic=30499.msg2031282#msg2031282

Or the Allais antigravitational effect:

https://www.theflatearthsociety.org/forum/index.php?topic=30499.msg760382#msg760382

Once that "fifth force" is activated (a very strong electrical field, sound, double torsion) it becomes much stronger than the force of terrestrial gravity.


"Well gentlemen, we will undertake this, although my conviction is strong that we shall prove only that the earth rotates on its axis, a conclusion which I think we may be said to be sure of already."

A. Michelson

This is the formula published by Michelson in 1904 and 1925:

(http://www.conspiracyoflight.com/Michelson-Gale_webapp/image004.png)

However, this is the CORIOLIS EFFECT FORMULA for circuital light beams.

Here it is:

Δt = 4AΩsinΦ/c^2 (where Φ is the latitude)

https://www.ias.ac.in/article/fulltext/pram/087/05/0071

The same formula was derived by Dr. Ludwik Silberstein in 1921:

https://www.theflatearthsociety.org/forum/index.php?topic=30499.msg2068289#msg2068289

That is, the CORIOLIS EFFECT upon the light beams is totally related to the closed contour area.

It is a physical effect, it deflects the path of the light beams.

By constrast, the rotational Sagnac effect is an electromagnetic effect, it modifies the speed of the light beams.

Michelson and Gale ONLY recorded the Coriolis effect of the ether drift upon the light beams; no rotational Sagnac effect was detected.

Here is the derivation of the rotational Sagnac effect:

https://www.theflatearthsociety.org/forum/index.php?topic=30499.msg2070082#msg2070082

https://www.theflatearthsociety.org/forum/index.php?topic=30499.msg2070907#msg2070907

The Sagnac effect formula is proportional to the radius of the Earth, while the Coriolis effect is proportional to the area of the interferometer and thus thousands of times smaller in magnitude than the Sagnac effect.

All this is really simple to understand, if they exists.
One, or more of these effects have to be activated? I suppose that's possible.

 However, these forces all appear to be constant. They would act the same, have the same observable effect no matter location and direction of projectile. The mathematical calculations are, or would be, a constant on a FE with or without ether drift.

This is not what's observable in the real world.

The trajectory of projectile drift changes exponentially as do the mathematical calculations that predict their path and target location on a RE with spin depending on location and direction.

Add together the reality that we've physically circumnavigated the globe, that with all the navigational calculations it takes to do so, we've seen with our own eyes the Coriolis effect, calculated this effect with math, the conclusion is irrefutable.

You seem to quote a lot of stuff that has little or nothing to do with the subject at hand.

Here's an example.

 "Biefeld-Brown antigravitational effect."

This from another forum

"I'm pretty sure this is just ionising air (stripping off some of the electrons from the N2, for example), and driving them through the electrical field between the strips of Aluminum foil; your TV does a similar thing when it take a cathode-ray (a beam of electrons) and uses electrical and magnetic fields to bend the beam in such a way to draw pictures on your screen...

The easy way to prove that it is just atmospheric ionisation, and not "anti-gravity", is to put one of these things in a vacuum chamber; if there is "anti-gravity", it should produce lift, otherwise it will just sit there...

That's not to say something like this is completely unfeasiable; Dr. Leik Myrabo has done some (real engineering) work on an air ionisation drive with the power supplied by ground-based lasers. I don't know that it would work (or be even useful), but it is theoretically possible...

http://www.eng.rpi.edu/mane/lightcraft/Curriculum/TAVD/intro/images/ship2-big.jpg">
 


 
Title: Re: Earths Curvature and spin effect on long range ballistics.
Post by: Curiosity File on October 02, 2018, 05:55:35 PM
Sandokhan
Here's some of the jargon you try to use as an example of something that effects ballistics.
As you can see it take a specific chemical composition to get this effect, which still is only theoretical. "aluminum plate" being one. Not used in artillery shells or bullets that I'm aware of. Please don't post a bunch of jargon about aluminum alloy composite casings because that's not part of the projectiles.

Again, even if this effect exists and had any impact on ballistics it would be a constant no matter which direction of projectile and locations involved as explained before.

http://jnaudin.free.fr/lifters/ekpsim/index.htm
 

 
Title: Re: Earths Curvature and spin effect on long range ballistics.
Post by: sandokhan on October 02, 2018, 06:32:30 PM
"I'm pretty sure this is just ionising air (stripping off some of the electrons from the N2, for example), and driving them through the electrical field between the strips of Aluminum foil; your TV does a similar thing when it take a cathode-ray (a beam of electrons) and uses electrical and magnetic fields to bend the beam in such a way to draw pictures on your screen...

The easy way to prove that it is just atmospheric ionisation, and not "anti-gravity", is to put one of these things in a vacuum chamber; if there is "anti-gravity", it should produce lift, otherwise it will just sit there..."


You haven't done your homework on the subject.

But I have.

(https://image.ibb.co/h8dhwz/bb1.jpg)
(https://image.ibb.co/dWCaGz/bb2.jpg)
(https://image.ibb.co/dMYMpK/bb3.jpg)

Biefeld-Brown Effect

Dr. Paul Biefeld used to do Einstein's homework while they were studying at the  Polytechnic University in Zurich in 1900.

Dr. Paul Biefeld was a classmate of A. Einstein.

 “Yes,” Biefeld told the Denison campus newspaper, “when Einstein would forget to go to a class, he would come and borrow my notes to get caught up on what he had missed."

Experimental proof that the vacuum consists of ether.

https://youtu.be/CGN65lse5yE

(vacuum test performed by Gravitec, increasing the voltage from 15kv to 18 kv, clear movement/thrust of the capacitor can be seen; near the end the power is switched off, and then turned on again, and we can the visible thrust of the capacitor for a second time)

The first video supplied by Gravitec in 2003:

https://www.theflatearthsociety.org/forum/index.php?topic=30499.msg1852363#msg1852363 (vacuum test #1, Biefeld-Brown effect part I, contains the experiments performed by T. Brown in oil)

https://youtu.be/ZE7Go7ptBRY

One of the best videos which exemplifies the Biefeld-Brown effect in vacuum:

https://web.archive.org/web/20050216062907/http://www-personal.umich.edu/~reginald/liftvac.html


http://lifters.online.fr/lifters/ascvacuum/index.htm (includes all necessary technical information and the video itself)


At the pressure of 1.72 x 10^-6 Torr ( High Vacuum conditions ), the apparatus rotates when the High Voltage is increased from 0 to +45 KV.


Dr. Takaaki Musha
Advanced Space Propulsion Investigation Committee (ASPIC)
Research Engineer on Naval Systems, Technical Research & Development Institute
Honda R&D Institute, Biefeld-Brown effect experiments

http://jnaudin.free.fr/lifters/musha/Musha.pdf

Explanation of dynamical Biefeld-Brown Effect from the standpoint of ZPF field

In 1956, T.T. Brown presented a discovery known as the Biefeld-Bown effect (abbreviated B-B effect) that a sufficiently charged capacitor with dielectrics exhibited unidirectional thrust in the direction of the positive plate.

From the 1st of February until the 1st of March in 1996, the research group of the HONDA R&D Institute conducted experiments to verify the B-B effect with an improved experimental device which rejected the influence of corona discharges and electric wind around the capacitor by setting the capacitor in the insulator oil contained within a metallic vessel . . . The experimental results measured by the Honda research group are shown . . .

. . . The theoretical analysis result suggests that the impulsive electric field applied to the dielectric material may produce a sufficient artificial gravity to attain velocities comparable to chemical rockets.


https://web.archive.org/web/20120710005059/http://www.ovaltech.ca/pdfss/Theoretical_Explanation_of_the_Biefield-Brown_Effect.pdf

Experiments carried out at the HONDA R&D Institute confirm that the Biefeld-Brown effect is real.

Direct experimental proof that the ether propagates through vacuum, causing the antigravitational Biefeld-Brown effect.

No gas left in the experiments: they were performed in vacuum.

Calculations indicate that ionic wind is at least three orders of magnitude too small to explain the magnitude of the observed force on the capacitor (in open air experiments).

Also the experiments carried out at the Honda R&D eliminated the possibility of ionic winds.
Title: Re: Earths Curvature and spin effect on long range ballistics.
Post by: Curiosity File on October 02, 2018, 06:40:40 PM
sandokhan
I go to your links, they take me to your web pages. I briefly study the information.
I click on more links, they take me to more stuff YOU post. I click on other links that finally takes me to places where maybe the original information comes from?
You use this info, take it to your post and try to apply it to FET. Successfully, maybe in some cases.

However sticking to the subject at hand, I don't see any relation this information of laser tech and bending of light etc. has anything to do with or effects the trajectory of a solid object.
Also I'll repeat the fact that all of that shows a constant effect which would not change the direction of a a projectile as seen with change of location and direction with real world effect.     
 
Title: Re: Earths Curvature and spin effect on long range ballistics.
Post by: Curiosity File on October 02, 2018, 06:52:29 PM
What does "ionic wind" have to with the subject at hand?
Title: Re: Earths Curvature and spin effect on long range ballistics.
Post by: Curiosity File on October 02, 2018, 07:58:50 PM
snadokhan this is the stuff you're trying to apply as evidence to confirmation of some kind kind of effect on ballistic. It does not in any way have anything to do with ballistics and the Coriolis effect.
Trying to sound intelligent by posting a plethora of scientific jargon, theoretical physics, hypothetical theories that have nothing to do with the subject at hand only proves you are trying to, for lack of better words, run a con job. This is in no way going to make me take you serious.

From this point I probably wont read another thing you post.

However here is some of the scientific jargon claim effects ballistic.
So take any one little aspect of this info and show me in a very simple explanation of how this might correlate to a small lead projectile traveling only 1,000 yards.
         

https://arxiv.org/ftp/arxiv/papers/1801/1801.06778.pdf

https://ntrs.nasa.gov/archive/nasa/casi.ntrs.nasa.gov/20100000021.pdf

https://phys.org/news/2018-02-ionic.html
Title: Re: Earths Curvature and spin effect on long range ballistics.
Post by: Humble B on October 02, 2018, 08:40:19 PM
snadokhan this is the stuff you're trying to apply as evidence to confirmation of some kind kind of effect on ballistic. It does not in any way have anything to do with ballistics and the Coriolis effect.

A formula by itself doesn't prove anything as long as the formula is not used, and the outcome of the calculation matches the observation and measurement. The proof of the pudding is in the eating. I would like to see sandokhan solving this question with his theory and formula's:

We have 2 canons, one on 30°0′N and the second exactly 50 miles north of the first one
The two canons are pointed at each other and loaded with a shell that will rotate with 20,000 r/minute.
They are aimed to hit each other with their shell, but the Cannoneers did not yet take the Coriolis effect in account because they do not know how to calculate it.

Now can you, sandokhan, show us how you can help them with your formula's and calculate for them how many inches or feet they have to aim their canons to the right, or left (or up or down) to get a direct hit.
Title: Re: Earths Curvature and spin effect on long range ballistics.
Post by: nickrulercreator on October 02, 2018, 10:26:53 PM
Sandokhan, you keep referencing the ether. As far as I know, the ether has no scientific evidence to support its existence, and in fact has been proven to not exist. But, by referencing it, you must have some evidence that supports its existence, right? May I see it?
Title: Re: Earths Curvature and spin effect on long range ballistics.
Post by: Curiosity File on October 02, 2018, 11:13:52 PM
Sandokhan, you keep referencing the ether. As far as I know, the ether has no scientific evidence to support its existence, and in fact has been proven to not exist. But, by referencing it, you must have some evidence that supports its existence, right? May I see it?

The problem with information sandokhan shares is it's basically antique and like the Ether drift theory has been disproved by modern equipment, technology and higher educated scientist.
All of the experiments, tests, hypothesis theories, formulas, etc come from 50 to 400 years ago.

The Ether Drift theory has been disprove multiple times and to my knowledge had never been replicated by any other scientific experiment throughout history other than the one that was flawed in the first place.

An Explanation of Dayton Miller’s
 Anomalous “Ether Drift” Result

https://arxiv.org/vc/physics/papers/0608/0608238v2.pdf

 
Title: Re: Earths Curvature and spin effect on long range ballistics.
Post by: nickrulercreator on October 03, 2018, 12:46:27 AM
Sandokhan, you keep referencing the ether. As far as I know, the ether has no scientific evidence to support its existence, and in fact has been proven to not exist. But, by referencing it, you must have some evidence that supports its existence, right? May I see it?

The problem with information sandokhan shares is it's basically antique and like the Ether drift theory has been disproved by modern equipment, technology and higher educated scientist.
All of the experiments, tests, hypothesis theories, formulas, etc come from 50 to 400 years ago.

The Ether Drift theory has been disprove multiple times and to my knowledge had never been replicated by any other scientific experiment throughout history other than the one that was flawed in the first place.

An Explanation of Dayton Miller’s
 Anomalous “Ether Drift” Result

https://arxiv.org/vc/physics/papers/0608/0608238v2.pdf

So what is his opinion on all of this? Surely he must acknowledge what science has shown to be true and false.
Title: Re: Earths Curvature and spin effect on long range ballistics.
Post by: sandokhan on October 03, 2018, 07:25:29 AM
As I said, you simply haven't done your homework on the subject.

The debunking of Roberts' catastrophic "analysis" of Dayton Miller's work:

https://www.theflatearthsociety.org/forum/index.php?topic=30499.msg751624#msg751624 (part I)

https://www.theflatearthsociety.org/forum/index.php?topic=30499.msg945952#msg945952 (part II)

The Ether Drift theory has been disprove multiple times and to my knowledge had never been replicated by any other scientific experiment throughout history other than the one that was flawed in the first place.

Dr. Yuri Galaev's super ether drift analysis, the most comprehensive ever performed:

https://www.theflatearthsociety.org/forum/index.php?topic=30499.msg1722791#msg1722791

Moreover, E.T. Whittaker proved the existence of ether (longitudinal waves) a long time ago:

https://www.theflatearthsociety.org/forum/index.php?topic=30499.msg1994059#msg1994059

The Aharonov-Bohm effect is another proof of the existence of ether (potential):

(https://image.ibb.co/c0CeLd/ton1.jpg)

Topological considerations of the Aharonov-Bohm effect:

https://www.theflatearthsociety.org/forum/index.php?topic=30499.msg1999598#msg1999598
Title: Re: Earths Curvature and spin effect on long range ballistics.
Post by: stack on October 03, 2018, 07:47:10 AM
sandokhan, simple question, with all that you have offered so far, can you apply your formulas and solve Humble B's puzzle?
Title: Re: Earths Curvature and spin effect on long range ballistics.
Post by: sandokhan on October 03, 2018, 09:10:23 AM
We have 2 canons, one on 30°0′N and the second exactly 50 miles north of the first one
The two canons are pointed at each other and loaded with a shell that will rotate with 20,000 r/minute.
They are aimed to hit each other with their shell, but the Cannoneers did not yet take the Coriolis effect in account because they do not know how to calculate it.


Here is the formula for the Coriolis deflection of a projectile:

http://www.dtic.mil/dtic/tr/fulltext/u2/a010816.pdf

Use it appropiately.

Title: Re: Earths Curvature and spin effect on long range ballistics.
Post by: Humble B on October 03, 2018, 12:49:02 PM
Here is the formula for the Coriolis deflection of a projectile:

http://www.dtic.mil/dtic/tr/fulltext/u2/a010816.pdf

Use it appropiately.

No, I asked you to use it. I'm too dumb to work with such complicated formula's on my own, and I do not understand the theory behind the formula.
But you seem to understand it all, and I may presume you checked the formulas.
Therefore I would like to see your calculations first, to learn from you how it should be done appropriately.
Title: Re: Earths Curvature and spin effect on long range ballistics.
Post by: Humble B on October 03, 2018, 01:09:33 PM
Here is the formula for the Coriolis deflection of a projectile:

http://www.dtic.mil/dtic/tr/fulltext/u2/a010816.pdf

But wait...... this is the formula which is derived from the theory about a projectile that flies over the surface of a spinning ball.

What I was asking for was a calculation with a formula derived from the theory about a projectile that flies over a stationary earth while it is subjected to a force created by a drifting aether. If you change the theory, you also have to change the formulas, because formula's always have to be explained and supported by the underlying theory.

Explaining the Coriolis force by a drifting aether, but then calculating the effect with the help of a spinning globe doesn't make sense.
Title: Re: Earths Curvature and spin effect on long range ballistics.
Post by: sandokhan on October 03, 2018, 02:48:50 PM
But wait...... this is the formula which is derived from the theory about a projectile that flies over the surface of a spinning ball.

What I was asking for was a calculation with a formula derived from the theory about a projectile that flies over a stationary earth while it is subjected to a force created by a drifting aether.


You do not seem to be very familiar with this type of debate.

The Coriolis force involves ROTATION: whether it be the rotation of the Earth, or the rotation of the ether field above the surface of the Earth.

The formulas are the same.

That is why the Coriolis effect, or Foucault's pendulum, cannot be used to debate heliocentrism vs. geocentrism.

You must understand Mach's principle, as it applies exactly to this situation.

By maintaining the relativity of all motion, especially rotational motion, E. Mach denied the existence of absolute motion and of absolute space. Accordingly, Mach maintained the equivalence of the Ptolemaic and the Copernican systems and the equivalence of rotating-system/fixed-universe and universe-rotating/fixed-system situations.

Mach's Principle: A body experiences no inertial forces when it is at rest or in uniform motion with respect to the center of mass of the entire universe. When its motion is nonuniform (accelerated) with respect to the total mass of the universe, it experiences forces such as centrifugal force and the Coriolis effect. Hence, the "local" behavior of matter is influenced by the "global" properties of the universe, i.e., those properties that describe the universe as a whole, which are studied in cosmology.

The Lense-Thirring effect as a consequence of Mach's Principle:

http://www.answers.com/topic/mach-s-principle

H. Thirring observed that the complete equivalence between the reference frames, explaining such phenomena as the Foucault pendulum equally well in a geocentric reference frame, is secured by definition by Einstein's 1915 work: "the required equivalence appears to be guaranteed by the general co-variance of the field equations." That is, Einstein's field equations are structured to supply the necessary upward force on the geosynchronous satellite in a geocentric as well as a heliocentric framework. Thus, H. Thirring notes that: "...in an Einsteinian gravitational field, caused by distant rotating masses, forces appear which are analogous to the centrifugal and Coriolis forces."

Max Born in his famous book,"Einstein's Theory of Relativity", Dover Publications,1962, pgs. 344 & 345 says:

"...Thus we may return to Ptolemy's point of view of a 'motionless earth'...One has to show that the transformed metric can be regarded as produced according to Einstein's field equations, by distant rotating masses. This has been done by Thirring. He calculated a field due to a rotating, hollow, thick-walled sphere and proved that inside the cavity it behaved as though there were centrifugal and other inertial forces usually attributed to absolute space.

Thus from Einstein's point of view, Ptolemy and Corpenicus are equally right."

Einstein himself also says:

"The struggle, so violent in the early days of science, between the views of Ptolemy and Copernicus would then be quite meaningless. Either CS could be used with equal justification. The two sentences, 'the sun is at rest and the earth moves,' or 'the sun moves and the earth is at rest,' would simply mean two different conventions concerning two different CS. -- Einstein and Infeld, The Evolution of Physics, p.212 (p.248 in original 1938 ed.)"


Therefore, distant rotary masses can cause local inertial forces, like the Coriolis and centrifugal forces, which perfectly mimic the inertial effects of a spinning Earth . This implies that there are two possible explanations for the inertial forces whenever objects are in relative rotational motion.

Mach's principle has been confirmed in theory by Hans Thirring and no experimental test has ever disproved this principle of relative motion.

The experiment performed by J. Barbour and B. Bertotti proved that a large hollow sphere (representing the distant star fields) rotating around a small solid sphere inside (modeling the Earth) produced exactly the same pattern of Coriolis and centrifugal forces that are claimed as proof of Earth's spinning in space. If the hollow shell of matter accelerates or rotates, any object inside the shell will tend to be carried along with the acceleration or rotation to some extent. There have arisen some questions re: the Lagrangian used by Barbour and Bertotti and also about the coordinate transformations discussed in their article, but the main experiment showed, quite clearly that Mach's Principle is correct.

http://www.freelists.org/post/geocentrism/Overview-Barbour-Bertotti

Ernst Mach proposed that it is the weight of the stars circling the Earth that drags Foucault pendulums around, creates Coriolis forces in the air that give the cyclones to our weather etc. Barbour and Bertotti (Il Nuovo Cimento 32B(1):1-27, 11 March 1977) proved that a hollow sphere (the universe) rotating around a solid sphere inside (the Earth) produced exactly the same results of Coriolis forces, dragging of Foucault pendulums etc. that are put forward as 'proofs' of heliocentricity.


Do you understand now?

The Coriolis effect is just a physical effect.

To distinguish between heliocentrism and geocentrism one needs the SAGNAC EFFECT.

Title: Re: Earths Curvature and spin effect on long range ballistics.
Post by: Humble B on October 03, 2018, 04:22:11 PM

Do you understand now?

Yes, now I understand that you do not understand your own theory yourself, and that you try to hide that by burying those who question it under mountains of irrelevant scientific and pseudo-scientific wrangling and studies of which only people with a PhD in Quantum Physics and General Relativity can tell if they make any sense or not.

When you know that "The Coriolis effect is just a physical effect" of one body moving over the surface of an other body, you ought to understand that just plain classical mechanics is sufficient to explain and calculate this Coriolis effect, and that referring to all those non-related studies you are referring to only serves the goal of blurring the picture to hide the truth that your "drifting aether theory" did never surpass the low level of useless phantasies.

Because in physics the difference between accepted scientific theories and useless phantasies is that the first ones have delivered a bunch of new formula's that can be used and tested and have proven to be reliable and useful in the real world of real physics. While the latter, the useless phantasies, never give birth to new formulas that can be tested and used to prove the phantasy more than just a phantasy.

That's why that as long as your "spinning aether phantasy" does not deliver any new formula's that successfully can and will be used to calculate the Coriolis force, or the torque and precession of a spinning gyroscope without the help of a spinning ball, your phantasies will never be accepted as theory, but always be mocked as a useless phantasy haunting the dark caverns of junk science.
Title: Re: Earths Curvature and spin effect on long range ballistics.
Post by: sandokhan on October 03, 2018, 04:35:45 PM
That's why that as long as your "spinning aether phantasy" does not deliver any new formula's that successfully can and will be used to calculate the Coriolis force, or the torque and precession of a spinning gyroscope, your phantasies will never be accepted as theory, but always be mocked as a useless phantasy haunting the dark caverns of junk science.

I am sorry to disappoint you.

Here is one of the most sought-after formulas, which no other physicist, not even Nobel prize winners, was able to derive it.

But I was.

The generalized Sagnac effect formula:

https://www.theflatearthsociety.org/forum/index.php?topic=30499.msg2070082#msg2070082

https://www.theflatearthsociety.org/forum/index.php?topic=30499.msg2070907#msg2070907

(https://image.ibb.co/dbZ7Kd/gsac2.jpg)
Title: Re: Earths Curvature and spin effect on long range ballistics.
Post by: Humble B on October 03, 2018, 04:53:03 PM
Great, I love to be disappointed.

Now disappoint me again by demonstrating how you use this formula:

(https://image.ibb.co/dbZ7Kd/gsac2.jpg)

to solve this problem without any help of a spinning earth:

We have 2 canons, one on 30°0′N and the second exactly 50 miles north of the first one
The two canons are pointed at each other and loaded with a shell that will rotate with 20,000 r/minute.
They are aimed to hit each other with their shell, but the Cannoneers did not yet take the Coriolis effect in account because they do not know how to calculate it.
Show us how you use this formula and calculate for them how many inches or feet they have to aim their canons to the right, or left (or up or down) to get a direct hit.
Title: Re: Earths Curvature and spin effect on long range ballistics.
Post by: Curiosity File on October 03, 2018, 05:37:04 PM
Sandokhan this is, well, wow, your really smart.
"Here is one of the most sought-after formulas, which no other physicist, not even Nobel prize winners, was able to derive it. But I was."

You should be able to solve Humble B's puzzle with a simple formula and explanation that an 8th grader can understand

Great, I love to be disappointed.

Now disappoint me again by demonstrating how you use this formula:

(https://image.ibb.co/dbZ7Kd/gsac2.jpg)

to solve this problem without any help of a spinning earth:

We have 2 canons, one on 30°0′N and the second exactly 50 miles north of the first one
The two canons are pointed at each other and loaded with a shell that will rotate with 20,000 r/minute.
They are aimed to hit each other with their shell, but the Cannoneers did not yet take the Coriolis effect in account because they do not know how to calculate it.
Show us how you use this formula and calculate for them how many inches or feet they have to aim their canons to the right, or left (or up or down) to get a direct hit.
Title: Re: Earths Curvature and spin effect on long range ballistics.
Post by: sandokhan on October 03, 2018, 06:02:25 PM
to solve this problem without any help of a spinning earth:

We have 2 canons, one on 30°0′N and the second exactly 50 miles north of the first one
The two canons are pointed at each other and loaded with a shell that will rotate with 20,000 r/minute.
They are aimed to hit each other with their shell, but the Cannoneers did not yet take the Coriolis effect in account because they do not know how to calculate it.
Show us how you use this formula and calculate for them how many inches or feet they have to aim their canons to the right, or left (or up or down) to get a direct hit.


That's easy.

You apply the following well-known results:

Therefore, distant rotary masses can cause local inertial forces, like the Coriolis and centrifugal forces, which perfectly mimic the inertial effects of a spinning Earth . This implies that there are two possible explanations for the inertial forces whenever objects are in relative rotational motion.

Mach's principle has been confirmed in theory by Hans Thirring and no experimental test has ever disproved this principle of relative motion.

The experiment performed by J. Barbour and B. Bertotti proved that a large hollow sphere (representing the distant star fields) rotating around a small solid sphere inside (modeling the Earth) produced exactly the same pattern of Coriolis and centrifugal forces that are claimed as proof of Earth's spinning in space. If the hollow shell of matter accelerates or rotates, any object inside the shell will tend to be carried along with the acceleration or rotation to some extent. There have arisen some questions re: the Lagrangian used by Barbour and Bertotti and also about the coordinate transformations discussed in their article, but the main experiment showed, quite clearly that Mach's Principle is correct.

http://www.freelists.org/post/geocentrism/Overview-Barbour-Bertotti

Ernst Mach proposed that it is the weight of the stars circling the Earth that drags Foucault pendulums around, creates Coriolis forces in the air that give the cyclones to our weather etc. Barbour and Bertotti (Il Nuovo Cimento 32B(1):1-27, 11 March 1977) proved that a hollow sphere (the universe) rotating around a solid sphere inside (the Earth) produced exactly the same results of Coriolis forces, dragging of Foucault pendulums etc. that are put forward as 'proofs' of heliocentricity.

Then you simply use the same well-known formula to solve the problem you posed.

http://www.dtic.mil/dtic/tr/fulltext/u2/a010816.pdf

If you disagree, you are going to have to disprove Mach's principle.
Title: Re: Earths Curvature and spin effect on long range ballistics.
Post by: juner on October 03, 2018, 06:30:36 PM
Sandokhan this is, well, wow, your really smart.
"Here is one of the most sought-after formulas, which no other physicist, not even Nobel prize winners, was able to derive it. But I was."

You should be able to solve Humble B's puzzle with a simple formula and explanation that an 8th grader can understand

Refrain from low-content posting in the upper fora. Warned.
Title: Re: Earths Curvature and spin effect on long range ballistics.
Post by: Curiosity File on October 03, 2018, 06:45:41 PM
Sandokhan this is, well, wow, your really smart.
"Here is one of the most sought-after formulas, which no other physicist, not even Nobel prize winners, was able to derive it. But I was."

You should be able to solve Humble B's puzzle with a simple formula and explanation that an 8th grader can understand


Refrain from low-content posting in the upper fora. Warned.

Warning excepted.
Sandakhan you've shown a high level of intelligence and education..
Please post a formula that solves Humble B's puzzle that I, with only a high school education from 45 years ago, can understand. 
Title: Re: Earths Curvature and spin effect on long range ballistics.
Post by: Curiosity File on October 03, 2018, 06:55:10 PM
to solve this problem without any help of a spinning earth:

If you disagree, you are going to have to disprove Mach's principle.

Again you are post stuff, "Mach's Principle",  that is nothing more than a hypothesis that the distant stars have some kind of influence on mass.
Distant stars don't exist on a flat earth with a dome.

Mach doesn't even prove his principle. It's it's just a hypothesis.


Is Mach's Principle Wrong?
Ask Question
up vote
27
down vote
favorite
23

This question was prompted by another question about a paper by Woodward (not mine). IMO Mach's principle is very problematic (?wrong) thinking. Mach was obviously influenced by Leibniz. Empty space solutions in GR would result in a Minkowski metric and would suggest no inertia. Mach's principle seems incompatible with GR. Gravitational waves could also be a problem. I had thought that papers like one by Wolfgang Rindler had more or less marginalised the Mach Principle, but I see lots of Internet discussion of it. Is it correct? Wrong? Is there evidence? (frame dragging experiments)?
Let's use this definition from ScienceWorld.Wolfram.com:

    In his book The Science of Mechanics (1893), Ernst Mach put forth the idea that it did not make sense to speak of the acceleration of a mass relative to absolute space. Rather, one would do better to speak of acceleration relative to the distant stars. What this implies is that the inertia of a body here is influenced by matter far distant.

https://physics.stackexchange.com/questions/5483/is-machs-principle-wrong

 
Title: Re: Earths Curvature and spin effect on long range ballistics.
Post by: sandokhan on October 03, 2018, 07:05:25 PM
Please post a formula that solves Humble B's puzzle that I, with only a high school education from 45 years ago, can understand.

I already did, even twice already.

http://www.dtic.mil/dtic/tr/fulltext/u2/a010816.pdf

Please try and understand.

You are attempting to use the Coriolis force as some kind of an argument to prove heliocentricity.

And you cannot do that: it cannot be done.

The Coriolis force, involving rotation, is a valid argument for BOTH heliocentrists and geocentrists.

Mach's principle, which has been shown to be true by both Thirring and Born,  shows that "the two sentences, 'the sun is at rest and the earth moves,' or 'the sun moves and the earth is at rest,' would simply mean two different conventions concerning two different CS".

One valid formula to be used by both heliocentrists and geocentrists.

The Coriolis effect is a physical effect.

To distinguish between heliocentricity and geocentricity you need the Sagnac effect, namely my formula, the best ever in the field:

(https://image.ibb.co/dbZ7Kd/gsac2.jpg)
Title: Re: Earths Curvature and spin effect on long range ballistics.
Post by: Curiosity File on October 03, 2018, 07:09:08 PM
Please post a formula that solves Humble B's puzzle that I, with only a high school education from 45 years ago, can understand.

I already did, even twice already.

http://www.dtic.mil/dtic/tr/fulltext/u2/a010816.pdf

Please try and understand.

You are attempting to use the Coriolis force as some kind of an argument to prove heliocentricity.

And you cannot do that: it cannot be done.

The Coriolis force, involving rotation, is a valid argument for BOTH heliocentrists and geocentrists.

Mach's principle, which has been shown to be true by both Thirring and Born,  shows that "the two sentences, 'the sun is at rest and the earth moves,' or 'the sun moves and the earth is at rest,' would simply mean two different conventions concerning two different CS".

One valid formula to be used by both heliocentrists and geocentrists.

The Coriolis effect is a physical effect.

To distinguish between heliocentricity and geocentricity you need the Sagnac effect, namely my formula, the best ever in the field:

(https://image.ibb.co/dbZ7Kd/gsac2.jpg)

I'm sorry, I made same mistake Humble B did. I forgot to add "without using the spinning ball" formula.
Title: Re: Earths Curvature and spin effect on long range ballistics.
Post by: juner on October 03, 2018, 07:29:02 PM
I think sandokhan's point is pretty clear. If you all want to argue the validity of it, go ahead, but stop asking the same question over and over when it has been answered. It is clear that sandokhan is saying that there is no difference in observed Coriolis whether you are using his model, or the RE model, therefore the equation is the same. Again, feel free to debate/argue the validity of his position.
Title: Re: Earths Curvature and spin effect on long range ballistics.
Post by: sandokhan on October 03, 2018, 08:12:07 PM
I forgot to add "without using the spinning ball" formula.

The Coriolis force exists only when one uses a rotating reference frame.

You don't need a speeding ball: the rotating distant stars or the rotating ether drift will do.

That is why a physical effect, just like the Coriolis force or Foucault's pendulum, cannot be used to distinguish between heliocentrism and geocentrism.

The RE used to have a mighty weapon in this debate: the Michelson-Gale experiment (1925).

A very simple formula.

You have the measured fringe shifts, the area of the interferometer, the speed of light; then, the angular velocity of the Earth can be found.

The proponents of aether theory have tried to argue saying that the MGX measures the flow of ether, but they were helpless when faced with the fact that Michelson claimed that the formula presented by him in 1925 is actually the Sagnac effect formula.

Since the Sagnac effect measures both linear/uniform/translational and rotational motion, there was nothing (up until this year) the proponents of aether theory could do.

This has always been the easiest way for heliocentrists to win each and every debate since 1925: simply use the MGX formula to justify that the Earth rotates around its own axis.

This is the reason I fought so hard since the summer of 2017 to prove that the formula used by Michelson is actually the Coriolis effect formula.

But that wasn't nearly enough.

I had to produce the CORRECT SAGNAC FORMULA, something no one else has been able to do since 1913, not Michelson, Lorentz, Post, nor Langevin, not anybody else.

(https://image.ibb.co/dbZ7Kd/gsac2.jpg)
Title: Re: Earths Curvature and spin effect on long range ballistics.
Post by: Tom Bishop on October 03, 2018, 08:43:56 PM
Question for Sandokhan: Where can we see controlled experimental evidence of the Focault Pendulum or the Coriolis Effect?

Focault Pendulum has mixed results (https://wiki.tfes.org/Foucault_Pendulum). There is no control. I have never seen a control for the Coriolis Effect, either.

When I speak of CONTROL, I am referring to, for example, a test where someone shoots a rifle at a target Eastwards, observes drift, and then shoots at a target Westwards, where the bullet drifts to the opposite direction. That tests the matter more conclusively than seeing some drift in one direction. My concern is that in your work you are taking these phenomena for granted, without the proper evidence that they exist at all.

The last few times we have talked about the need for controlled experimentation, you seemed to dismiss the need entirely. I would like to see your sources that these phenomena have been properly tested.

As we know, Round Earthism is full of people who will make deceptive conclusions form bad science to get the result they need for their government funding, or to maintain their belief system. As far as I am concerned, if there is no control, it is trash science and should not be granted any credibility or consideration.
Title: Re: Earths Curvature and spin effect on long range ballistics.
Post by: Mysfit on October 03, 2018, 08:52:34 PM
It took me all of 30 seconds, but I found a video to, hopefully, help Tom

https://www.youtube.com/watch?v=jX7dcl_ERNs

It was the first one that came up when i searched "coriolis effect, bullet shot east and west"
Title: Re: Earths Curvature and spin effect on long range ballistics.
Post by: sandokhan on October 03, 2018, 08:55:58 PM
Foucault's pendulum can easily be debunked:

https://www.theflatearthsociety.org/forum/index.php?topic=30499.msg944125#msg944125

However, the Coriolis effect on light beams is real, and so is its formula.

As for the Coriolis effect on projectiles, especially bullets, if that is a deceptive conclusion, so much the better.

But you cannot assume this to be a fact: you need to address the best case scenario for the RE, just like I have always done.

And this is how it's done: you need a rotating frame of reference to put to rest the RE's claims re: the Coriolis force.

Remember it is not the Coriolis effect you have to worry about, but the conclusions of the Michelson-Gale experiment. If you want to argue that the MGX was not performed properly, or that the equation they used is not the correct one, you need to produce/prove the correct formula in order to address this issue.
Title: Re: Earths Curvature and spin effect on long range ballistics.
Post by: Tom Bishop on October 03, 2018, 09:00:24 PM
It took me all of 30 seconds, but I found a video to, hopefully, help Tom

https://www.youtube.com/watch?v=jX7dcl_ERNs (https://www.youtube.com/watch?v=jX7dcl_ERNs)

It was the first one that came up when i searched "coriolis effect, bullet shot east and west"

On the East target:

(https://i.imgur.com/mSnBGDV.png)

On the West target:

(https://i.imgur.com/0Zl4kWr.png)

I do not see that the bullets drifted to the opposite direction. On the Western trial the bullets just seem to drop downwards, with bullet holes appearing very slightly to both the left and right sides of the target.

How does this support or demonstrate a Round Earth model?
Title: Re: Earths Curvature and spin effect on long range ballistics.
Post by: sandokhan on October 03, 2018, 09:11:28 PM
The author of the video is confusing the Coriolis effect (a horizontal deflection) with the Eotvos effect (a vertical displacement).

If he is shooting due east or due west, he is going to measure the Eotvos effect, and not the Coriolis effect.

Please find a video which measures the Coriolis effect in both directions north-south, south-north.
Title: Re: Earths Curvature and spin effect on long range ballistics.
Post by: sandokhan on October 03, 2018, 09:33:54 PM
Two viewers have correctly pointed out the same thing:

This isn't actually due to Coriolis, and he did not properly explain the Coriolis Effect anyway. The Coriolis Effect is due to a difference in the speed of rotation of the Earth at two differing points of latitude. The rotation speed of the Earth is greater at the Equator than it is at the poles. The Earth isn't rotating beneath the bullet. The rotation of the Earth is imparted to the bullet. Conservation of momentum. The bullet shifts because the Earth is rotating more slowly, or more quickly, at the destination latitude than the firing latitude.

What he is explaining here is the Eötvös Effect. It is also due to the rotation of the Earth, and also varies with latitude, but it is caused by the centrifugal force that is imparted to the bullet by the Earth's rotation, which is why it is apparent in East-West shooting.


This explanation is wrong. Coriolis effect does not effect a bullet east west, only north south. The Coriolis effect comes from the Change in relative speed of the earth, relative to an object in motion above it. As a bullet fired north from the equator flies, it is travelling both north at 3000 feet per second, and east at 1500 feet per second, a target north of the equator is also traveling east, but at a slightly slower speed, since the bullet is moving both north and east, but east faster than the target, it will, in the same time, travel further east than the target does while in air, striking to the right. In reverse, if you fire from the slower moving target south towards the equator, the target at the equator is moving faster east than the bullet, so it the bullet fired this direction will also strike right. When firing from the equator south, the bullet is traveling east faster and will hit left (in both cases, when firing from a faster moving point on earth, to a slower one, the bullet strikes further east, when slower to faster, further west.)

When firing East / West, a different thing happens, the shooter and the target are moving at the same speed. So when in flight, the target does not actually move up and towards the bullet, when firing west, or down and away when shooting east as depicted in the video. This is just plain 100% false, not arguments about it. Something else is happening to cause the results shown above. When firing east, the bullet is moving in the same direction as the earth, so, relative to the center of the earth, it is moving at a higher degree of rotation around the fixed point in terms of degrees per hour. This causes a greater amount of centrifugal force, which "slightly" counteracts the effect of gravity, essentially making the bullet "lighter" and it will follow a flatter arc, causing a higher "hit". When firing west, the angular velocity or speed of rotation around a fixed point is lower, causing less centrifugal force, and allowing gravity to take more effect, essentially making the bullet heavier, causing a lower hit. This is the Eotvos effect, something tangentially related, but different than the Coriolis effect.


Now, someone should call the author of the video and ask if the bullets were fired EXACTLY due east or due west. If that is the case, the slight horizontal deflections were caused by the wind.

However, if the author of the video says that the bullets were not fired exactly due east or west, then the RE have a huge problem: as has been pointed out, on the western trial the bullets just drop downwards, no Coriolis effect has been measured.
Title: Re: Earths Curvature and spin effect on long range ballistics.
Post by: Curiosity File on October 03, 2018, 10:02:49 PM
It took me all of 30 seconds, but I found a video to, hopefully, help Tom

https://www.youtube.com/watch?v=jX7dcl_ERNs (https://www.youtube.com/watch?v=jX7dcl_ERNs)

It was the first one that came up when i searched "coriolis effect, bullet shot east and west"

On the East target:

(https://i.imgur.com/mSnBGDV.png)

On the West target:

(https://i.imgur.com/0Zl4kWr.png)

I do not see that the bullets drifted to the opposite direction. On the Western trial the bullets just seem to drop downwards, with bullet holes appearing very slightly to both the left and right sides of the target.

How does this support or demonstrate a Round Earth model?
It is clear to me the east target they hit high high of the center, with exeption of one that hits near center but not bellow.
To the wast they low of the center.
If there was only gravitational pull they would always hit low.
This is physical proof that to the east the target drops and to the wast the target raises. 
Title: Re: Earths Curvature and spin effect on long range ballistics.
Post by: Tom Bishop on October 03, 2018, 10:57:07 PM
- The vertical distance drift is not complimentary. The Eastern shots drift vertically nowhere near as much as the Western shots.

- We can see that two of his Eastern shots are lined up with the target vertically, whereas none of the Western shots are.

- The author of the video actually says that the shots were low because he was shooting at a mirage.
Title: Re: Earths Curvature and spin effect on long range ballistics.
Post by: Humble B on October 03, 2018, 11:50:21 PM
Therefore, distant rotary masses can cause local inertial forces, like the Coriolis and centrifugal forces

No, that's impossible. Maybe distant rotary masses can cause forces, but not inertial like the Coriolis and centrifugal force.

Q- Why not?
A- Because inertial forces like the Coriolis force and the centrifugal force do not exist. I repeat: They do NOT EXIST. They are so called "fictitious forces" or in plane English: "fake forces". A bullet flying over a rotating surface does not move in a curved trajectory, no matter what the surface underneath the flying bullet is doing, it will always go in a straight line (only bend downwards by gravity) And when the bullet is following a straight trajectory there is no such force as a Coriolis force pushing it left or right. The curved line observed in the trajectory of a bullet which we attribute to the Coriolis effect is an optical illusion.

This optical illusion is caused by a rotating reference frame and an observer who is rotating with this reference frame. And because the observer and his reference frame are all rotating in a curved direction while the flying bullet is not, the observer will see the straight trajectory of the bullet as curved.

Here in this video a computer simulation of the Coriolis effect showing clearly the straight trajectory of the moving object and the optical illusion of a curved line:

https://www.youtube.com/watch?v=lG3snOEis0w


When the theory of sandokhan would be true, and the surface of the earth does not rotate but the projectiles are pushed into a curved trajectory by a force, we can not do that with an inertial or fictitious force (because non-existing forces can not change the motion of objects with mass)
When that force would come from external rotating masses, then we need a real force, a force that can be detected and measured and used to create new and better formula's to calculate the effect of that force on bodies with mass.

Maybe sandokhan has found those formula's, but apparently he is still searching for a way how to use them on flying objects with mass.
Title: Re: Earths Curvature and spin effect on long range ballistics.
Post by: Curiosity File on October 04, 2018, 01:41:57 AM
There's a number of things we are all in agreement of.
#1 there's a force that causes a projectile to hit to the right(to the east) of the target when fired from the equator northward in northern hemisphere.
#2 there is no lateral deflection in either direction shooting to the west or to the east.

I think this is absolute proof that the earth is not a spinning disc.

#3 We all agree we can see the sky moving from east to west.
This is absolute poof something is moving.

 The sky it what's moving and everything in it, the sun the moon the stars , and this what FET claims, and this is what sandokhan claims is the force that pushes a bullet off target as it's moving one direction. Sandokhan also claims that it's this force that pushes down on the bullet that's flying east and not so much flying west.

So we've established, hypothetically, that there is a force in the sky pushing from east to west.
Wouldn't that push the bullet to the left(west) fired to the north?

Some FET say the earth moves too. Sandokhan indicated that the earth is moving from west to east, which increases or decreases the "downward force from the sky", depending on direction,(east or west), you're shooting.

I can't see how on a FE with a dome and a wall encompassing it with everything in the sky no more than 3,000 miles away spinning in a circle,(not circling the earth), allow the earth to move anywhere. It would have to be two different objects not connected to each other allowing one to move separate one another.

Also if they were separate one another, and moving in opposite direction, as verified, hypothetically, by sandokhan, the forces would cancel each other.

Now to really throw mud on the works is move to the southern atmosphere and everything reverses direction in regards to the bullet.
Also what seems to be the most mind boggling thing to the FE believers, and what they don't have an answer to, is the fact the bullet ALWAYS hits to the right of the target no mater shooting to the north from the south or from the south to the north on the northern hemisphere, and again reverses when you move to the southern hemisphere.   

   


 
Title: Re: Earths Curvature and spin effect on long range ballistics.
Post by: Tom Bishop on October 04, 2018, 01:44:18 AM
There's a number of things we are all in agreement of.
#1 there's a force that causes a projectile to hit to the right(to the east) of the target when fired from the equator northward in northern hemisphere.
#2 there is no lateral deflection in either direction shooting to the west or to the east.

Where did you prove any of that?
Title: Re: Earths Curvature and spin effect on long range ballistics.
Post by: Curiosity File on October 04, 2018, 01:52:34 AM
There's a number of things we are all in agreement of.
#1 there's a force that causes a projectile to hit to the right(to the east) of the target when fired from the equator northward in northern hemisphere.
#2 there is no lateral deflection in either direction shooting to the west or to the east.

Where did you prove any of that?

You know exactly what it proved.

I'm sorry Tom I misread your question.

Those two particular quotes are stated as everybody, at least on this thread, have agreed that this takes place and is physically observable in the real world.

If you disagree with that tell us why.

Title: Re: Earths Curvature and spin effect on long range ballistics.
Post by: stack on October 04, 2018, 02:00:39 AM
There's a number of things we are all in agreement of.
#1 there's a force that causes a projectile to hit to the right(to the east) of the target when fired from the equator northward in northern hemisphere.
#2 there is no lateral deflection in either direction shooting to the west or to the east.

Where did you prove any of that?

Just for the record, sniper ballistics software takes into account the Coriolis effect. The U.S. Army and Marine Corps use handheld ballistic computers (PDA’s) loaded with Horus Vision targeting software. The software takes into account, as well as many other factors, the Coriolis Effect as a part of its calculations.

"These formulas take in all the factors governing bullet flight from the point of launch to target strike. Included amongst these are bullet weight, shape, and drag contributors, muzzle velocity, rifling twist rate and direction (as viewed from the chamber end of the barrel). Atmospheric considerations such as air density, humidity and range-wind are addressed, along with Earth ‘rotation effects’ (coriolis) and, of course, the location and behavior of the target."

https://www.horusvision.com/download/manual_Horus_ATrag-v385.pdf
Title: Re: Earths Curvature and spin effect on long range ballistics.
Post by: Curiosity File on October 04, 2018, 03:11:52 AM
There's a number of things we are all in agreement of.
#1 there's a force that causes a projectile to hit to the right(to the east) of the target when fired from the equator northward in northern hemisphere.
#2 there is no lateral deflection in either direction shooting to the west or to the east.

Where did you prove any of that?

Just for the record, sniper ballistics software takes into account the Coriolis effect. The U.S. Army and Marine Corps use handheld ballistic computers (PDA’s) loaded with Horus Vision targeting software. The software takes into account, as well as many other factors, the Coriolis Effect as a part of its calculations.

"These formulas take in all the factors governing bullet flight from the point of launch to target strike. Included amongst these are bullet weight, shape, and drag contributors, muzzle velocity, rifling twist rate and direction (as viewed from the chamber end of the barrel). Atmospheric considerations such as air density, humidity and range-wind are addressed, along with Earth ‘rotation effects’ (coriolis) and, of course, the location and behavior of the target."

https://www.horusvision.com/download/manual_Horus_ATrag-v385.pdf
Thanks stack for bringing these facts up.
I not only have friends and family the have been and still are in the military from as far back as my dad being the gunners mate in WWII. Served on battle ships as well smaller vessels.
I have read a lot of ballistics material over many years and I have a great deal knowledge and understanding of the complicated formulas needed to calculate trajectories of non propulsion projectiles as well as missiles with their own propulsion systems.
Thanks for the link
     
Title: Re: Earths Curvature and spin effect on long range ballistics.
Post by: Tom Bishop on October 04, 2018, 03:59:24 AM
Just for the record, sniper ballistics software takes into account the Coriolis effect. The U.S. Army and Marine Corps use handheld ballistic computers (PDA’s) loaded with Horus Vision targeting software. The software takes into account, as well as many other factors, the Coriolis Effect as a part of its calculations.

"These formulas take in all the factors governing bullet flight from the point of launch to target strike. Included amongst these are bullet weight, shape, and drag contributors, muzzle velocity, rifling twist rate and direction (as viewed from the chamber end of the barrel). Atmospheric considerations such as air density, humidity and range-wind are addressed, along with Earth ‘rotation effects’ (coriolis) and, of course, the location and behavior of the target."

https://www.horusvision.com/download/manual_Horus_ATrag-v385.pdf

Please provide actual evidence that this effect actually exists. What you have provided is not actual evidence. It is not actual evidence because you have provided no experimental evidence that this effect exists, or that this software accurately predicts it.

The U.S. Marine Corps Sniping Manual does not make mention of the Coriolis Effect at all. Snipers are not taught to account for it.

U.S. Marine Corps Sniping Manual

https://archive.org/details/milmanual-fmfm-1-3b-sniping-u.s.-marine-corps/page/n0

The sniper must know the general principles of: perspective, vanishing point, perspective drawing, delineation, and geographical areas of intelligence operations. However, the words "Coriolis" or "Coriolis Effect," do not appear anywhere in the U.S. Marine Corps Sniping Manual.
Title: Re: Earths Curvature and spin effect on long range ballistics.
Post by: stack on October 04, 2018, 04:36:58 AM
Just for the record, sniper ballistics software takes into account the Coriolis effect. The U.S. Army and Marine Corps use handheld ballistic computers (PDA’s) loaded with Horus Vision targeting software. The software takes into account, as well as many other factors, the Coriolis Effect as a part of its calculations.

"These formulas take in all the factors governing bullet flight from the point of launch to target strike. Included amongst these are bullet weight, shape, and drag contributors, muzzle velocity, rifling twist rate and direction (as viewed from the chamber end of the barrel). Atmospheric considerations such as air density, humidity and range-wind are addressed, along with Earth ‘rotation effects’ (coriolis) and, of course, the location and behavior of the target."

https://www.horusvision.com/download/manual_Horus_ATrag-v385.pdf

Please provide actual evidence that this effect actually exists. What you have provided is not actual evidence. It is not actual evidence because you have provided no experimental evidence that this effect exists, or that this software accurately predicts it.

From the US Army Material Command Ballistics Research Laboratories

“The Production of Firing Tables for Cannon Artillery"

Table H, pg 103

“Corrections to Range, in Meters, to Compensate for the Rotation of Earth"

(https://i.imgur.com/vhCKXgL.png)

http://www.dtic.mil/dtic/tr/fulltext/u2/826735.pdf
Title: Re: Earths Curvature and spin effect on long range ballistics.
Post by: Tom Bishop on October 04, 2018, 04:44:03 AM
Where are the artillery experiments with and without this very slight adjustment?

There is no evidence. You are posting heresy and hypothesis from an paper called "Production of Firing Tables for Cannon Artillery" which gives predictions for various situations and their associated assumptions, not evidence.

From the introduction of your paper:

Quote
Ideally, a firing table enables the artilleryman to solve his fire problem and to hit the target with the first round fired. In the present state of the art, this goal is seldom achieved, except coincidentally. The use of one or more forward observers, in conjunction with the use of a firing table, enables the artilleryman to adjust his fire and hit the target with the third or fourth round fired.

Enough said.

The famous astronomer Tycho Bahe and his research organization, which was the largest astronomical organization of his time, financed with 5% of the federal budget of the Danish Government, conducted artillery experiments and found no effect due to the rotation of the earth.

Astronomer Giovanni Riccioli describes here:

http://arxiv.org/ftp/arxiv/papers/1012/1012.3642.pdf

Quote
VIII. Tycho also argues that if the cannon experiment were performed at the
poles of the Earth, where the ground speed produced by the diurnal motion is
diminished, then the result of the experiment would be the same regardless of
toward which part of the horizon the cannon was fired. However, if the experiment
were performed near the equator, where the ground speed is greatest, the result
would be different when the ball is hurled East or West, than when hurled North or
South.

The form of the argument is thus: If Earth is moved with diurnal motion, a ball fired
from a cannon in a consistent manner would pass through a different trajectory when hurled
near the poles or toward the poles, than when hurled along the parallels nearer to the Equator,
or when hurled into the South or North. But this is contrary to experience. Therefore, Earth is
not moved by diurnal motion.

If Tycho is to be believed, experiments have shown this to be correct. Moreover,
if a ball is fired along a Meridian toward the pole (rather than toward the East or
West), diurnal motion will cause the ball to be carried off [i.e. the trajectory of the
ball is deflected], all things being equal: for on parallels nearer the poles, the ground
moves more slowly, whereas on parallels nearer the equator, the ground moves more
rapidly.7

The Copernican response to this argument is to deny it, or to concede it but claim
that the differences in trajectory fall below our ability to measure. But in fact the
argument is strong, and this response is not.

Riccioli concludes with:

Quote
None of the above examples of what should happen if the Earth moves are in
accord with what we see. Therefore, the Earth does not move with diurnal, much less
annual, motion.
Title: Re: Earths Curvature and spin effect on long range ballistics.
Post by: Curious Squirrel on October 04, 2018, 05:14:41 AM
Where are the artillery experiments with and without this very slight adjustment?

There is no evidence. You are posting heresy and hypothesis, not evidence.

The famous astronomer Tycho Bache and his research organization, which was the largest astronomical organization of his time, financed with 5% of the federal budget of the Danish Government, conducted artillery experiments and found no effect due to the rotation of the earth.

Astronomer Giovanni Riccioli describes here:

http://arxiv.org/ftp/arxiv/papers/1012/1012.3642.pdf

Quote
VIII. Tycho also argues that if the cannon experiment were performed at the
poles of the Earth, where the ground speed produced by the diurnal motion is
diminished, then the result of the experiment would be the same regardless of
toward which part of the horizon the cannon was fired. However, if the experiment
were performed near the equator, where the ground speed is greatest, the result
would be different when the ball is hurled East or West, than when hurled North or
South.

The form of the argument is thus: If Earth is moved with diurnal motion, a ball fired
from a cannon in a consistent manner would pass through a different trajectory when hurled
near the poles or toward the poles, than when hurled along the parallels nearer to the Equator,
or when hurled into the South or North. But this is contrary to experience. Therefore, Earth is
not moved by diurnal motion.

If Tycho is to be believed, experiments have shown this to be correct. Moreover,
if a ball is fired along a Meridian toward the pole (rather than toward the East or
West), diurnal motion will cause the ball to be carried off [i.e. the trajectory of the
ball is deflected], all things being equal: for on parallels nearer the poles, the ground
moves more slowly, whereas on parallels nearer the equator, the ground moves more
rapidly.7

The Copernican response to this argument is to deny it, or to concede it but claim
that the differences in trajectory fall below our ability to measure. But in fact the
argument is strong, and this response is not.

Riccioli concludes with:

Quote
None of the above examples of what should happen if the Earth moves are in
accord with what we see. Therefore, the Earth does not move with diurnal, much less
annual, motion.
An yet every long range shooter is taught how to take both the Coriolis Effect, and the Eötvös effect into account for firing of ballistic weapons. The horizontal difference of a shot fired at a pole (where the Coriolis effect is strongest) is in the range of 4 inches for a shot at 1000 yards. Why continue with this teaching if there's no such thing to account for? Is this another group that is 'in' on the conspiracy? The distances done in most of these experiments fall well within the error range of the ability to measure the change as I recall the last time this was brought up. But sure, claiming the expected movement isn't an appreciable amount at the distances tested 'isn't a strong case' is where you prefer to lay your chips. Sounds good.

Maybe they are taught to account for some slight effect. We tends to describe the Coriolis Effect as a result of slight gravitational pull from the moving stars in some of our models.

But the effects and maths Tycho and Riccioli discuss are different and should be more pronounced if the earth were rotating, and the analysis directly asses the concept of a rotating earth and what it should produce.
Why? They discuss the same effect. The distance being by necessity much short, and the object being fired much larger. I see nothing about either experiment that require their conclusions, rather than the fact the distances and etc they are using will not produce a noticeable enough effect for instruments at that time. Let me see if I can dig up the source and info I used last time you posted this.

EDIT: Ah, here we are. Apologies, this doesn't reference Tycho specifically, but it does mention cannon experiments in general and how they are wholly inadequate and too inaccurate to measure the effect. http://adsabs.harvard.edu/full/1913PA.....21..208R

I would also once again point out Tycho would have had an inherent bias in regards to wanting to believe/prove the Earth doesn't rotate due to beliefs at the time.
I'll just post this again. I would direct you in particular to page 211 and the section marked '2. Impeded Fall' as the best location for an experiment that repeatedly showed a deflection concurrent with the Coriolis force, to within a probability error of within 0.03. As well, before Tom can crow about it again, the notation in there discussing 'no experimental proof' is clearly referring to the claimed S/N drift they also tested for, but whose probability error was close to 0.10, outside a suitable confidence interval to claim they had measured it.
Title: Re: Earths Curvature and spin effect on long range ballistics.
Post by: sandokhan on October 04, 2018, 05:32:05 AM
Because inertial forces like the Coriolis force and the centrifugal force do not exist.

Completely wrong.

The author of this statement has no knowledge of the original set of J.C. Maxwell's equations, which do include the Coriolis term/vorticity.

Dr. Frederick Tombe explains:

http://gsjournal.net/Science-Journals/Research%20Papers-Mathematical%20Physics/Download/6212 (Wikipedia and Coriolis Force)

http://gsjournal.net/Science-Journals/Essays-Miscellaneous/Download/5288 (The Centrifugal Force and the Coriolis Force)

http://gsjournal.net/Science-Journals/Research%20Papers-Mechanics%20/%20Electrodynamics/Download/3842 (The Coriolis Force)

"The Coriolis force is not what it is said to be in modern textbooks. It is not a fictitious illusion which is merely a product of making an observation from a rotating frame of reference. The cyclones in the atmosphere are observable absolutely, and they arise from two very real compound inertial forces, and these inertial forces have a physical cause which is closely related to, but not identical to, the magnetic forces."

http://gsjournal.net/Science-Journals/Research%20Papers-Astrophysics/Download/3161 (The Coriolis Force in Maxwell’s Equations)

Title: Re: Earths Curvature and spin effect on long range ballistics.
Post by: Tom Bishop on October 04, 2018, 06:50:54 AM
I'll just post this again. I would direct you in particular to page 211 and the section marked '2. Impeded Fall' as the best location for an experiment that repeatedly showed a deflection concurrent with the Coriolis force, to within a probability error of within 0.03. As well, before Tom can crow about it again, the notation in there discussing 'no experimental proof' is clearly referring to the claimed S/N drift they also tested for, but whose probability error was close to 0.10, outside a suitable confidence interval to claim they had measured it.

The author you champion does nothing except throw away experiments that do not give him the result he wants.

From the article:

Quote
Guglielmini at Bolgona in 1790 was the first to experiment with bodies falling from a height. He used lead balls accurately turned and polished. He suspended each one by a thread attached to that point of the ball which was on top when it was floated in mercury. He cut the thread with a knife and allowed the ball to fall 90 feet. Unfortunately it was only after six months that he suspends a plumb line at the place in order to find the deviation of the balls from the vertical. He neither tells us from what point of the compass the threads were cut, nor the bearing of the sides of the tower.

Benzenberg repeated these experiments at Hamburg in 1802, using generally a fall of 235 feet. He observed during the day time, while his predecessor worked after midnight. But he took the precaution to suspend his plumbline immediately before and after each set of falls, and to cut the thread half of the time from the north and the other half from the south. The median was found by means of a compass needle. The balls were an alloy of lead and zinc. As the extreme deviations of his balls from the vertical are nine times the mean distance of all and, as according to his own statement, the sun, coming out of the clouds at noon, upon one occasion, warmed the south side of his wooden tower so much as to throw his plumb line 1.5 lines towards the north, Benzenberg's results, like those of Guglielmini, cannot be considered to offer even a qualitative proof of the earth's rotation. [Read: "He said documented something odd that happened on one occasion, which changed a marker that he could see moving, so that means that all of the trials are invalid!!"]

P. 210

Benzenberg also dropped 40 balls in a mine at Schebusch from a height of 262 Paris feet, but gives the results of only 28 of these. Gilbert says that neither of these two attempts of Benzenberg's have any scientific value. [Read: "I need to give no reason, it's obviously an invalid experiment!"]

...

Hooke's trials before the Royal Society in London in 1680 have only an historic value. He affirmed that the fall of a heavy body would be more to the south than to the east. As the height used was only 27 feet and the theoretical easterly deviation less than half a millimeter, Gilbert declares him to be under an illusion. [Read: Ignoring that Hooke was a Royal Astronomer and knew what the results should be, his experiments dismissed because it just sounds like too small of a drop]

Similar judgments must be meted out to other minor experimenters, whose names and results are scarcely worth mentioning. [ie. "I judge a whole range of experiments which I will not even describe here to be insufficient!"]

Dismissing experiments for some made up reason is the very definition of bias. In fact, cherry-picking what you want to see, without mentioning the results of the scores of experiments you dismiss, makes you a liar.

Quote
EDIT: Ah, here we are. Apologies, this doesn't reference Tycho specifically, but it does mention cannon experiments in general and how they are wholly inadequate and too inaccurate to measure the effect.

This was likely said by this obviously biased author because there are no cannon experiments which support a rotating earth, only ones which do not support it.

Any cannon that can shoot straight should experience deviation due to the rotation of the earth; and the cannon can be shot in multiple directions to identify any inherent left or right deviation error of the cannon. Multiple directions and multiple trials should easily show the difference between a biased cannon and the rotation of the earth. The deviation due to earth rotation should change depending on direction.

We have been told many times that the Coriolis Effect does apply to cannons. So where are these experiments which support your model?

Quote
I would also once again point out Tycho would have had an inherent bias in regards to wanting to believe/prove the Earth doesn't rotate due to beliefs at the time.

LOL Bias? Are you kidding? The author you picked to champion on this subject does nothing but throw away experiments which do not suit him. Just read your article! Where does Tycho show bias in his scientific pursuits?

Team Copernicus are the only biased crooks here.
Title: Re: Earths Curvature and spin effect on long range ballistics.
Post by: stack on October 04, 2018, 06:52:36 AM
"FM 4-15 Coast Artillery Field Manual, Seacoast Artillery, Fire Control and Position Finding 1943"

(https://i.imgur.com/hxcnuGw.jpg)

https://archive.org/details/Fm4-15/page/n371

Search on ‘rotation’ and you’ll find dozens more references to the rotation of earth.
Title: Re: Earths Curvature and spin effect on long range ballistics.
Post by: Tom Bishop on October 04, 2018, 07:04:32 AM
"FM 4-15 Coast Artillery Field Manual, Seacoast Artillery, Fire Control and Position Finding 1943"

https://i.imgur.com/hxcnuGw.jpg

https://archive.org/details/Fm4-15/page/n371

Search on ‘rotation’ and you’ll find dozens more references to the rotation of earth.

We know that there are a lot of people who think it exists and write about it. However, it would be a much stronger argument to show that there have been artillery experiments on reality to directly demonstrate the matter, rather than words about theory or supposition of what slight adjustments should be made.
Title: Re: Earths Curvature and spin effect on long range ballistics.
Post by: stack on October 04, 2018, 07:29:42 AM
"FM 4-15 Coast Artillery Field Manual, Seacoast Artillery, Fire Control and Position Finding 1943"

https://i.imgur.com/hxcnuGw.jpg

https://archive.org/details/Fm4-15/page/n371

Search on ‘rotation’ and you’ll find dozens more references to the rotation of earth.

We know that there are a lot of people who think it exists and write about it. However, it would be a much stronger argument to show that there have been artillery experiments on reality to directly demonstrate the matter, rather than words about theory or supposition of what slight adjustments should be made.

These aren't just people that simply think and write about it. They actually build devices to calculate for the Coriolis effect for long range ballistics, projectiles ranging 1000m or more.
So your logic is that military builds into their ranging devices calculations factoring for the rotation of the earth just because they "think it exists"? If that were an incorrect assumption on their part, I suspect they would never hit a target.

And btw, where are the experiments here, where's the evidence:

"Astronomer Giovanni Riccioli describes here:

http://arxiv.org/ftp/arxiv/papers/1012/1012.3642.pdf"

It's all supposition and hearsay. I'm providing 20th century military documentation that shows when it comes to long range artillery ballistics they definitely factor in the rotation of the earth. You're posting non-experimental 17th century stuff like, "Suppose that a very large cannon ball, weighing 60 or 80 pounds, traverses 250 paces in 2 human pulsebeats, or 2 seconds..." Nonsense. Get some real evidence.

Title: Re: Earths Curvature and spin effect on long range ballistics.
Post by: AATW on October 04, 2018, 09:29:09 AM
Dismissing experiments for some made up reason is the very definition of bias. In fact, cherry-picking what you want to see, without mentioning the results of the scores of experiments you dismiss, makes you a liar.
Dude, that's pretty much your MO. You do this all the time.
In a recent thread you dismissed the Cavendish experiment, a well-established, repeatable experiment, based on some article by a crazy bloke with no scientific credibility.
Title: Re: Earths Curvature and spin effect on long range ballistics.
Post by: totallackey on October 04, 2018, 10:45:59 AM

Do you understand now?

Yes, now I understand that you do not understand your own theory yourself, and that you try to hide that by burying those who question it under mountains of irrelevant scientific and pseudo-scientific wrangling and studies of which only people with a PhD in Quantum Physics and General Relativity can tell if they make any sense or not.
Yeah...errr...wait...what!?!?

You, on the one hand, decry sandokhan for his pertinent, informational posts, claiming,"...only people with a PhD in Quantum Physics and General Relativity can tell if they make sense or not.", while on the other having no such degree yourself!?!?!

WTF, dude!?!?


But wait, there is more:
When you know that "The Coriolis effect is just a physical effect" of one body moving over the surface of an other body, you ought to understand that just plain classical mechanics is sufficient to explain and calculate this Coriolis effect, and that referring to all those non-related studies you are referring to only serves the goal of blurring the picture to hide the truth that your "drifting aether theory" did never surpass the low level of useless phantasies.
You offer up judgment again having nothing to justify your supposed point.

Plus you cannot even correctly spell simple words, even with the advantage of spell check.
Because in physics the difference between accepted scientific theories and useless phantasies is that the first ones have delivered a bunch of new formula's that can be used and tested and have proven to be reliable and useful in the real world of real physics. While the latter, the useless phantasies, never give birth to new formulas that can be tested and used to prove the phantasy more than just a phantasy.
sandokhan has provided relevant topical content to this OP and your failure to understand it is not his problem.
That's why that as long as your "spinning aether phantasy" does not deliver any new formula's that successfully can and will be used to calculate the Coriolis force, or the torque and precession of a spinning gyroscope without the help of a spinning ball, your phantasies will never be accepted as theory, but always be mocked as a useless phantasy haunting the dark caverns of junk science.
Oh, I see...

In your world, qualified, vetted research and published scientific analysis is, ...junk science.

I think you are neither humble, nor can you be labeled a scientist.

Please consider changing your moniker.
Title: Re: Earths Curvature and spin effect on long range ballistics.
Post by: totallackey on October 04, 2018, 10:49:06 AM
Sandokhan this is, well, wow, your really smart.
"Here is one of the most sought-after formulas, which no other physicist, not even Nobel prize winners, was able to derive it. But I was."

You should be able to solve Humble B's puzzle with a simple formula and explanation that an 8th grader can understand


Refrain from low-content posting in the upper fora. Warned.

Warning excepted.
Sandakhan you've shown a high level of intelligence and education..
Please post a formula that solves Humble B's puzzle that I, with only a high school education from 45 years ago, can understand.
That maybe more difficult to perform than you are willing to accept.

You might want to enlist the help of a more educated friend to read the papers sandokhan has provided in order to get up to speed.

I am confident that approach can help you.
Title: Re: Earths Curvature and spin effect on long range ballistics.
Post by: Curious Squirrel on October 04, 2018, 01:10:39 PM
Tom you asked for experimental evidence showing the Coriolis effect. Just looking to present it. I presented a source that lists a number of them, please don't assume this means I'm 'championing' an author. Feel free to look those experiments up and vet them however you please. I'm sure people would love to discuss them with you here.

For the record most sources I'm finding DO agree with that author's simple statement of essentially 'the cannonball experiments of the late 1500's and early 1600's lacked the ability to measure the small deviation that would occur'. I also have yet to find anything on what Tycho did in this aspect beyond thought experiments and relying on secondary information. Most sources suggest the scientists of that time were expecting a much larger differential in part due to a lack of understanding of inertia.
Title: Re: Earths Curvature and spin effect on long range ballistics.
Post by: Mysfit on October 04, 2018, 01:30:58 PM
Hello Tom,
Thanks for watching the video.

- The vertical distance drift is not complimentary. The Eastern shots drift vertically nowhere near as much as the Western shots.

- We can see that two of his Eastern shots are lined up with the target vertically, whereas none of the Western shots are.
I like it when I can refute something complicated.
The west one has UA (I can't disprove UA) + Coriolis, making it very low.
The east one is lower due to UA but higher due to Coriolis, balancing out. Sorta.


- The author of the video actually says that the shots were low because he was shooting at a mirage.
Wouldn't both targets be at mirage distance? Do we account for this for only one target? That ain't scientific.

I read the US marine sniping manual you kindly provided and can confirm that it does not account for the Coriolis effect, but has some caveats.
- The manual covers 100-1,000 yards.
- "a hit anywhere on the body is normally disabling, so a small error is acceptable in combat"
Text version of the manual: https://archive.org/stream/pdfy-ITwQpfAWncRTv7rs/U.S.%20Marine%20Corps%20Sniper%20Manual_djvu.txt (https://archive.org/stream/pdfy-ITwQpfAWncRTv7rs/U.S.%20Marine%20Corps%20Sniper%20Manual_djvu.txt)

The targets in the video are on the outside edge of the range (1,000 yards) and the guy even says it is only of consequence at higher ranges.
Given that there is a few inches difference on the targets, I would consider the effect a waste of time to account for at shorter distances.
The heights are different with the only change being the direction. Something is happening and the current solution is the earth moving.
I don't know of a flat model that moves so... wait. Why isn't there one? I'll start a new topic.

Hope that helps, I will read up some more then check if there's a wiki page to edit.
Title: Re: Earths Curvature and spin effect on long range ballistics.
Post by: Tom Bishop on October 04, 2018, 02:21:01 PM
These aren't just people that simply think and write about it. They actually build devices to calculate for the Coriolis effect for long range ballistics, projectiles ranging 1000m or more.
So your logic is that military builds into their ranging devices calculations factoring for the rotation of the earth just because they "think it exists"? If that were an incorrect assumption on their part, I suspect they would never hit a target.

According to the Artillery paper you posted on the previous page they do not hit their targets!

http://www.dtic.mil/dtic/tr/fulltext/u2/826735.pdf

From the Introduction:

Quote
Ideally, a firing table enables the artilleryman to solve his fire problem and to hit the target with the first round fired. In the present state of the art, this goal is seldom achieved, except coincidentally. The use of one or more forward observers, in conjunction with the use of a firing table, enables the artilleryman to adjust his fire and hit the target with the third or fourth round fired.

So yes, this shows that what you are proposing absolutely needs demonstration.

Quote
http://arxiv.org/ftp/arxiv/papers/1012/1012.3642.pdf"

It's all supposition and hearsay. I'm providing 20th century military documentation that shows when it comes to long range artillery ballistics they definitely factor in the rotation of the earth. You're posting non-experimental 17th century stuff like, "Suppose that a very large cannon ball, weighing 60 or 80 pounds, traverses 250 paces in 2 human pulsebeats, or 2 seconds..." Nonsense. Get some real evidence.

You have it wrong. From the article: "If Tycho is to be believed, experiments have shown this to be correct."

Tycho performed experiments. What you have provided are not experiments.
Title: Re: Earths Curvature and spin effect on long range ballistics.
Post by: Tom Bishop on October 04, 2018, 02:58:26 PM
Given that there is a few inches difference on the targets, I would consider the effect a waste of time to account for at shorter distances.

Unless the sniper is 100% on all other hard-to-predict variables, a few inches can be the difference between a kill shot and a miss.

Arguing that snipers do not use Coriolis does because it is a 'waste of time' does not help the Round Earth argument that snipers account for Coriolis when sniping. That significantly weakens the argument that this effect actually exists.
Title: Re: Earths Curvature and spin effect on long range ballistics.
Post by: sandokhan on October 04, 2018, 03:21:16 PM
The Coriolis/Eotvos effects terms are a minor contribution to the calculations of the correct trajectory of a projectile (from a few centimeters to a few meters).

The biggest term, by far, the major contribution comes from the curvature equation.

This is the RE formula for a ballistic trajectory:

R = [vo2sin(2θo)]/g x {1 + [vo2/gRe][cos2θo]}

This is the FE formula for a ballistic trajectory (limit as Re goes to infinity):

R = [vo2sin(2θo)]/g

The derivation can be found here:

https://www.theflatearthsociety.org/forum/index.php?topic=30499.msg2032069#msg2032069 (part II, formula)

The difference amounts not to a few centimeters/few meters, but is in the range of kilometers.

This is the reason why no other FE has dared to touch this subject, not any of the youtube FE, or anybody else.

They simply cannot explain this phenomenon.

Unless...

The correct FE equation is brought into play:

R = [vo2sin(2θo)]/f(k)

k is the variable electrogravitational value, which depends on the altitude, the atmospheric ether tide, the density of ether at a certain altitude, and the spin rate

The curvature factor is ~EQUAL to the antigravitational effect produced by the spin rate of the projectile which forms a torsion field which partially cancels out the g force.

Title: Re: Earths Curvature and spin effect on long range ballistics.
Post by: AATW on October 04, 2018, 03:25:45 PM
Arguing that snipers do not use Coriolis does because it is a 'waste of time' does not help the Round Earth argument that snipers account for Coriolis when sniping. That significantly weakens the argument that this effect actually exists.
The effect is small, even over the sorts of distances snipers operate at
Some info here:

https://www.washingtoncitypaper.com/columns/straight-dope/article/13039128/do-snipers-compensate-for-the-earthrsquos-rotation-what-the-coriolis

It probably has less of an effect than other factors like the wind but there is no "argument" that it exists, it just does. It's not something which is a matter of opinion.
Title: Re: Earths Curvature and spin effect on long range ballistics.
Post by: juner on October 04, 2018, 04:06:22 PM
That maybe more difficult to perform than you are willing to accept.

You might want to enlist the help of a more educated friend to read the papers sandokhan has provided in order to get up to speed.

I am confident that approach can help you.

Tone it down, please. You have two posts in a row that attack a user directly. Please stick to attacking the claims/arguments, and keep the rants in AR. Warned.
Title: Re: Earths Curvature and spin effect on long range ballistics.
Post by: Tom Bishop on October 04, 2018, 04:38:55 PM
Quote
The Coriolis/Eotvos effects terms are a minor contribution to the calculations of the correct trajectory of a projectile (from a few centimeters to a few meters).

The biggest term, by far, the major contribution comes from the curvature equation.

]This is the RE formula for a ballistic trajectory:

R = [vo2sin(2θo)]/g x {1 + [vo2/gRe][cos2θo]}

This is the FE formula for a ballistic trajectory (limit as Re goes to infinity):

R = [vo2sin(2θo)]/g

The derivation can be found here:

https://www.theflatearthsociety.org/forum/index.php?topic=30499.msg2032069#msg2032069 (part II, formula)

The difference amounts not to a few centimeters/few meters, but is in the range of kilometers.

This is the reason why no other FE has dared to touch this subject, not any of the youtube FE, or anybody else.

They simply cannot explain this phenomenon.

Look at what we have been talking about.

Artillary, which is purported to require adjustments for the "Coriolis Effect" and other effects, is NOT ACCURATE.

From the 1967 Artillery paper (http://www.dtic.mil/dtic/tr/fulltext/u2/826735.pdf) we had read:

Quote
Ideally, a firing table enables the artilleryman to solve his fire problem and to hit the target with the first round fired. In the present state of the art, this goal is seldom achieved, except coincidentally. The use of one or more forward observers, in conjunction with the use of a firing table, enables the artilleryman to adjust his fire and hit the target with the third or fourth round fired.

This 1973 article on Firing Tables (http://www.dtic.mil/dtic/tr/fulltext/u2/909704.pdf) says:

Quote
When today's field artillery firing tables are used with today's approved delivery techniques [as described in VM 6-401], accurate fire can be brought to bear on targets.

Such a statement can only be made because today's approved delivery techniques recognize that many errors (both precision and bias errors) exist and those techniques arc designed to minimize these errors. The techniques are not designed to produce first round hits, nor does the statement above infer that such hits can be achieved.

This 2016 quote on the topic of modern artillery methods (https://www.quora.com/How-does-artillery-crew-calculate-target), from a claimed expert named Guy Schuchman, says that the same problems exist with today's modern improvements:

Quote
It's extremely rare for the first round to hit the target. It's just too much data which not all of it can be measured in 100% accuracy and human errors are quite common: small offsets in calculating the coordinates of the target or the gun, small errors in calibration, humidity of the explosive propellant, etc.. The first round is just a test round. When it falls near the target it's the artillery observer's job to see how far and in what offset did it hit away from the target and provide the FDC with the data.

A 2017 paper by Australia's Armament Research Service (https://www.icrc.org/en/download/file/36109/ares-icrc-report-indirect-fire.pdf) admits the same:

Quote
Even though great effort is made to calculate the effect of environmental and ballistic variables, an unguided artillery projectile will not reliably strike the exact point at which it is aimed. Although artillerymen strive for first round accuracy, this will still be measured in tens of meters, and in deliberate targeting or combat engagements this introduces a degree of uncertainty when assessing the safety of friendly forces and non-combatants. Properly employed, artillery gun and mortar projectiles and rockets land in a predictable area (accuracy) in a non-predictable fashion (precision), and in common with small arms fire (especially machine guns), the employment of artillery systems yields a ‘beaten zone’ or field of fire into which rounds will fall. This zone is generally cigar-shaped with the long axis falling along the line from the gun to the target, as deviation tends to occur in range rather than azimuth. The length and breadth of the zone is range dependent, as with greater range, external factors have more time to exert influence on the projectile flight.

Only after missing a number of times, and adjusting the alignment of the cannon, does artillery accurately hit its target.

Anyone who claims that artillery demonstrates the Coriolis Effect, or any other effect, will need to provide actual evidence. Many claimants of the Coriolis Effect are guilty of neglecting to provide actual evidence for their assertions.
Title: Re: Earths Curvature and spin effect on long range ballistics.
Post by: stack on October 04, 2018, 04:45:08 PM
These aren't just people that simply think and write about it. They actually build devices to calculate for the Coriolis effect for long range ballistics, projectiles ranging 1000m or more.
So your logic is that military builds into their ranging devices calculations factoring for the rotation of the earth just because they "think it exists"? If that were an incorrect assumption on their part, I suspect they would never hit a target.

According to the Artillery paper you posted on the previous page they do not hit their targets!

http://www.dtic.mil/dtic/tr/fulltext/u2/826735.pdf

From the Introduction:

Quote
Ideally, a firing table enables the artilleryman to solve his fire problem and to hit the target with the first round fired. In the present state of the art, this goal is seldom achieved, except coincidentally. The use of one or more forward observers, in conjunction with the use of a firing table, enables the artilleryman to adjust his fire and hit the target with the third or fourth round fired.

So yes, this shows that what you are proposing absolutely needs demonstration.

Quote
http://arxiv.org/ftp/arxiv/papers/1012/1012.3642.pdf"

It's all supposition and hearsay. I'm providing 20th century military documentation that shows when it comes to long range artillery ballistics they definitely factor in the rotation of the earth. You're posting non-experimental 17th century stuff like, "Suppose that a very large cannon ball, weighing 60 or 80 pounds, traverses 250 paces in 2 human pulsebeats, or 2 seconds..." Nonsense. Get some real evidence.

You have it wrong. From the article: "If Tycho is to be believed, experiments have shown this to be correct."

Tycho performed experiments. What you have provided are not experiments.

Show me one of Tycho Bahe's experiments. You keep claiming he did them, I can find no evidence that he did the "cannon" experiment or the like from the paper you posted or anywhere else.

Title: Re: Earths Curvature and spin effect on long range ballistics.
Post by: Tom Bishop on October 04, 2018, 05:30:39 PM
Followup: Sadokan seems to claim that the "curvature equation" is a more major contribution than the alleged Coriolis Effect.

Quote from: Sandokhan
The Coriolis/Eotvos effects terms are a minor contribution to the calculations of the correct trajectory of a projectile (from a few centimeters to a few meters).

The biggest term, by far, the major contribution comes from the curvature equation.

But where is evidence that they are even using a 'curvature equation' in artillery?

From the U.S. Army Research Laboratory we read the following from an artillery paper:

http://www.arl.army.mil/arlreports/2010/ARL-TR-5118.pdf

Quote
Projectile Flight Dynamics

A 6-DOF rigid projectile model is employed to predict the dynamics of a projectile in flight. These equations assume a flat Earth. The 6-DOF comprises the three translational components describing the position of the projectile’s center of mass and the three Euler angles describing the orientation of the projectile with respect to the Earth. Figures 1 and 2 provide a visualization of the degrees of freedom.

(https://i.imgur.com/Rxwbu47.png)

Another artillery paper:

http://www.dtic.mil/dtic/tr/fulltext/u2/a278426.pdf

Quote
(https://i.imgur.com/0peIurA.png)

From a paper on cannons and missile launch systems:

http://www.dtic.mil/dtic/tr/fulltext/u2/a278426.pdf

Quote
(https://i.imgur.com/142rSGq.png)

There are many other papers which detail how a Flat Earth is assumed in many of the military research papers, which I do not have the time to look for at the moment.

See this video: "Army, Air Force, CIA, Navy & NASA Documents Admit FLAT EARTH! (https://www.youtube.com/watch?v=tCwO1AHvWWI)"
Title: Re: Earths Curvature and spin effect on long range ballistics.
Post by: sandokhan on October 04, 2018, 05:48:59 PM
But where is evidence that they are even using a 'curvature equation' in artillery?

Here is the mathematical derivation of the trajectory equation for a spherical earth:

(http://image.ibb.co/fre6Qx/art1.jpg)
(http://image.ibb.co/et6syH/art2.jpg)
(http://image.ibb.co/hBYGQx/art3.jpg)

As for the 6-DOF equations, you need to search a little deeper:

https://www.diva-portal.org/smash/get/diva2:905698/FULLTEXT01.pdf (pg 9)

https://www.math.hmc.edu/~dyong/math164/2008/zitter/finalreport.pdf


The RE equation for the trajectory of projectile is real.

The FE equation, with a fixed g, is totally inaccurate.

That is why you need to take into account the ether drift, so that g becomes variable.

Using UA will get you nowhere: the RE will simply present the two equations side by side and win any debate.
Title: Re: Earths Curvature and spin effect on long range ballistics.
Post by: Tom Bishop on October 04, 2018, 05:55:20 PM
Sandokan, I linked to papers on .mil domains, and you linked me to papers on .edu and .org domains. The video I had linked to likewise has documents from .mil domains.

"Here is the mathematical derivation of the trajectory equation for a spherical earth" is not actual evidence. As a Flat Earther you should know that actual evidence is needed for these wild Round Earth claims.

At this point I have severe doubts that anyone in this thread even knows what evidence is. A school report called "Modeling Flight over a Spherical Earth (https://www.math.hmc.edu/~dyong/math164/2008/zitter/finalreport.pdf)" from a student at Harvey Mudd College is not actual evidence.
Title: Re: Earths Curvature and spin effect on long range ballistics.
Post by: stack on October 04, 2018, 05:57:58 PM
But where is evidence that they are even using a 'curvature equation' in artillery?

From the U.S. Army Research Laboratory we read the following from an artillery paper:

When I presented numerous military papers expressing the inclusion of the effect you wrote:

Where are the artillery experiments with and without this very slight adjustment?

There is no evidence. You are posting heresy and hypothesis from an paper called "Production of Firing Tables for Cannon Artillery" which gives predictions for various situations and their associated assumptions, not evidence.

Therefore, how does your inclusion of same type documents constitute the evidence you seek?

As well, where are those Tycho Bahe experiments you claim to exist?
Title: Re: Earths Curvature and spin effect on long range ballistics.
Post by: sandokhan on October 04, 2018, 06:01:56 PM
As a Flat Earther

Right.

I'm trying to point out to you that the mathematical derivation of the equation for the trajectory of a projectile is REAL.

It leads to two different equations: one RE, the other one FE.

The first one was used by the US Navy in WWI and WWII.

Please read:

http://www.eugeneleeslover.com/USN-GUNS-AND-RANGE-TABLES/OP-770-1.html

The RE equation is valid and accurate, while the FE equation with a fixed g is inaccurate.

Your UA theory will lead you nowhere.

That is why you need the ether drift to explain this crucial set of equations: by modifying g, your problem is solved.
Title: Re: Earths Curvature and spin effect on long range ballistics.
Post by: sandokhan on October 04, 2018, 07:48:32 PM
Here is a sample calculation using both formulas.

Let vo = 1,000 m/s and θo = 45 degrees (neglecting air drag and other factors such as the Coriolis and the Eotvos effects).

R = [1,0002sin90degrees][{1 + 1,0002cos2(45deg)}/(9.81x6,378,164)}]/9.81 = 102.7514 km

The curvature factor is: 1.007991

Even though f(k) is a nonlinear function of k, let's approximate this factor by k in order to get an estimate:

Rflat earth = 101.937 km (a difference of 814 meters)

102,751.4 = 1,0002sin90degrees/k

k = 9.73223


It is assumed by modern science that “differences in spin rate do not affect the aerodynamic coefficients”. (Ballistics 2011, 26th International Symposium, pg. 474)

“How far the bullet drops has nothing to do with spinning other than that it keeps it in the most favorable aerodynamic position”.

But spinning has everything to do with the calculation of the range, as proven by the experiments carried out by Dr. Bruce DePalma.

"The only explanation for this effect is that both balls are drawing energy into themselves from an unseen source, and the rotating ball is thus “soaking up” more of this energy than its counterpart – energy that would normally exist as gravity, moving down into the earth.

With the addition of torsion-field research we can see that the spinning ball was able to harness naturally spiraling torsion waves in its environment, which gave it an additional supply of energy."

https://depalma.pairsite.com/Absurdity/Absurdity05/SecretOfForceMachine.html

The original Force Machine was constructed in 1971, figure (1). The total weight of the apparatus was 276 lbs. The "active" mass at the rim of the flywheels was 10 lbs. The assembly was suspended from a spring scale and the gyroscopes driven counter-rotating at 7600 r.p.m. Under these conditions the support cylinder was driven at 4 r.p.s. to precess the gyros. A consistent set of experiments repeatably showed 4 - 6 lbs. of weight loss.

“In DePalma’s device, two magnetized gyroscopes were mounted side-by-side within a cylinder, (see next image,) spinning in opposite directions to each other — one clockwise and the other counterclockwise.

Both gyroscopes (here referred to as flywheels) were in the same position, with the bottom of their axles pointing straight downward and the top of their axles straight upward.

The cylinder that held these gyroscopes in place was then also made to rotate from its side, causing the axles of the gyroscopes to continually rotate end-over-end in the vertical plane like spokes in a spinning wheel.

Since the inertial forces generated by the gyroscopes cause them to naturally resist being moved out of position, even more aetheric energy could be harnessed by forcing them to do so. And as we said in the last chapter, defying gravity is as simple as gathering some of the downward-streaming aetheric energy and redirecting it off to the side, like the bending of a hose. This redirection can be accomplished by simple rotation.”

(https://divinecosmos.com/wp-content/uploads/2005/11/forcemachine01.gif)

“Dr. DePalma‘s “Force” machine would initially weigh 276 lbs. before being activated. The gyroscopes would be driven in counter-rotating directions at 7600 rpm each, and then the entire cylinder would be rotated or precessed at 4 revolutions per second. Any movement faster than this would create internal forces great enough to fracture the support axles for the gyroscopes, which would destroy the machine.

Once the Force Machine was running at this speed, it would repeatedly show 4-6 lbs. of weight loss!”


http://depalma.pair.com/gyrodrop.html (experiment carried out by the team of researchers who worked with Dr. Bruce DePalma)

Gyro Drop Experiment

In this experiment a fully enclosed, electrically driven gyroscope is released to fall freely under the influence of gravity. The elapsed time taken to fall a measured distance of 10.617 feet was measured, with the rotor stopped and also with the rotor spinning at approximately 15,000 RPM.

Data was gathered on a Chronometrics Digital Elapsed Dime Clock measuring 1/10,000 second, actuated by two phototransistor sensors placed in the paths of two light beams which were consecutively interrupted by the edge of the casing of the falling gyroscope.

A fully encased, spinning gyroscope drops faster than the identical gyroscope non-spinning, when released to fall along its axis.


The word “curvature” has to be replaced with the phrase “DePalma spinning effect” in the US Naval manual for curvature calculations applied to the range of the projectile:

https://i2.wp.com/mathscinotes.com/wp-content/uploads/2017/12/CurvatureTable1.png?resize=768%2C723 (Table of Curvatures for Different Horizontal Ranges)

Rotational bodies have different 'classical' principles such as:

- variable inertia

- variable gravitational acceleration either up or down (the spinning ball launch experiment)



https://www.theflatearthsociety.org/forum/index.php?topic=30499.msg753387#msg753387 (DePalma spinning ball experiments)

"The precise application of Newton’s laws … have to be restricted to non-rotating mechanical objects in field-free space. In a gravitational field, the possibility of extraction of greater energy by a new mechanical dimension [rotation] opens up the possibility of an anti-gravitational interaction"

Dr. Bruce DePalma, 1977


One of the greatest physicists in the world, Dr. Eugene Podkletnov, performed an amazing experiment.

(http://www.holoscience.com/wp/wp-content/uploads/2002/05/disc.jpg)


https://www.theflatearthsociety.org/forum/index.php?topic=30499.msg1833949#msg1833949 (part I)

A rapidly spinning disc of superconducting ceramic suspended in the magnetic field of three electric coils, was tested with a variety of materials and objects suspended above it, with measurable and consistent effects. In each case, the objects suspended above the rotating magnetic fields lightened by from 0.5 percent to 2 percent.

When Dr. Eugene Podkletnov increased the rotation speed to 20,000 rpm, the loss of weight became 5%. The highest speed that could be obtained was 30,000 rpm, where the loss of weight was 9% (figures offered by Dr. Podkletnov in the 2004 interview at the Tampere Technical University in Finland).

"In 1995, the Max Planck Institute of Physics did a follow up study, and was able to confirm the results."

(http://www.enterprisemission.com/images/anti-g6.jpg)


https://arxiv.org/pdf/hep-th/9505094.pdf


Impulse Gravity Generator Based on Charged Y Ba2Cu3O7-y Superconductor with Composite Crystal Structure

https://arxiv.org/pdf/physics/0108005.pdf

The observed phenomenon appears to be absolutely new and unprecedented in the
literature. It cannot be understood in the framework of general relativity.


Weak gravitation shielding properties of composite bulk Y Ba2Cu3O7-x superconductor
below 70 K under e.m. field

https://arxiv.org/pdf/cond-mat/9701074.pdf

A toroidal disk with an outer diameter of 275 mm and a thickness of 10 mm was prepared using conventional ceramic technology in combination with melt-texture growth. Two solenoids were placed around the disk in order to initiate the current inside it and to rotate the disk about its central axis. Samples placed over the rotating disk initially demonstrated a weight loss of 0.3-0.5%. When the rotation speed was slowly reduced by changing the current in the solenoids, the shielding effect became considerably higher and reached 1.9-2.1% at maximum.


Study of Light Interaction with Gravity Impulses and Measurements of the Speed of Gravity Impulses

https://www.researchgate.net/publication/281440634_Study_of_Light_Interaction_with_Gravity_Impulses_and_Measurements_of_the_Speed_of_Gravity_Impulses

An attempt has been made in this work to study the scattering of laser light by the gravity-like impulse produced in an impulse gravity generator (IGG) and also an experiment has been conducted in order to determine the propagation speed of the gravity impulse. The light attenuation was found to last between 34 and 48 ns and to increase with voltage, up to a maximum of 7% at 2000 kV. The propagation time of the pulse over a distance of 1211 m was measured recording the response of two identical piezoelectric sensors connected to two synchronized rubidium atomic clocks. The delay was 63±1 ns, corresponding to a propagation speed of 64c.


"Dr. Podkletnov also describes his “force beam generator” experiment in detail, and provides insights into improvements that he’s made over the last decade to increase the force produced by this experimental gravity-beam. The force beam is generated by passing a high-voltage discharge from a Marx-generator through a YBCO emitter suspended in a magnetic field, and Podkletnov has described it as being powerful enough to knock over objects in the lab, as well as capable of being tuned by even punch holes in solid materials.

Podkletnov maintains that a laboratory installation in Russia has already demonstrated the 4in (10cm) wide beam’s ability to repel objects a kilometre away and that it exhibits negligible power loss at distances of up to 200km."


G-acceleration is a VARIABLE whose value can be decreased in direct proportion to the amount of ether generated (by sound, by double torsion, by a high-electrical field).

As the temperature of the superconductor reaches the level of temperature of the ether, the subquark lattice state of the material will approach that of the baryon state of ether of the atom.

If the superconductor is being rotated at high speed, the Whittaker potential ether longitudinal strings will begin to form a torsion field which will act as a terrestrial gravity shield.

The gravity-beam created by Dr. Podkletnov is able to modify at a much higher speed than normal the ether quantum state of the target (allowing it to reach the baryon state of ether).
Title: Re: Earths Curvature and spin effect on long range ballistics.
Post by: Humble B on October 04, 2018, 08:52:57 PM
Because inertial forces like the Coriolis force and the centrifugal force do not exist.

Completely wrong.

The author of this statement has no knowledge of the original set of J.C. Maxwell's equations, which do include the Coriolis term/vorticity.

Dr. Frederick Tombe explains:

Not at all completely wrong.

The authors and papers you are referring to do not dispute the fact that the Coriolis force is an inertial force, and that the path of a bullet over the surface of the earth is straight, not curved.

My point was that if the earth is not spinning, and the bullets trajectory is curved, then this curve can not be the result of an "inertial force" as you claim, and the curved trajectory of the bullet above a stationary earth has nothing to do with the Coriolis effect.

Even if science would change their mind about the fictitious status of inertial forces, you (those who claim the physical the earth is not spinning) still need another force than an inertial one to prove your case, because above a stationary surface a curved trajectory has nothing to do with the Coriolis effect.


The RE equation is valid and accurate, while the FE equation with a fixed g is inaccurate.

Within science this is called: "Evidence for a spinning globe".

Title: Re: Earths Curvature and spin effect on long range ballistics.
Post by: sandokhan on October 04, 2018, 09:08:07 PM
Within science this is called: "Evidence for a spinning globe".

You will be disappointed again.

The FE equation with a fixed g is the WRONG formula.

g is a variable.

As such, we can immediately obtain the correct value for the entire calculation.


Spinning globe you say...

But there is no curvature whatsoever across lake Michigan.

 
Grand Haven Daily Tribune   April 3, 1925

COAST GUARDS SEE MILWAUKEE LIGHTS GLEAM

Captain Wm. J. Preston and Crew See Lights of Milwaukee

and Racine Clearly From Surf Boat

ANSWER TO FLARE

Crew Runs Into Lake in Search For Flashing Torch

Grand Haven Daily Tribune   April 3, 1925

Captain Wm. J. Preston and his U. S. Coast Guard crew at Grand Haven harbor witnessed a strange natural phenomenon last night, when they saw clearly the lights of both Milwaukee and Racine, shining across the lake.  As far as known this is the first time that such a freak condition has prevailed here.

 The phenomena was first noticed at shortly after seven o’clock last night, when the lookout called the keeper’s attention to what seemed to be a light flaring out on the lake.  Captain Preston examined the light, and was of the impression that some ship out in the lake was “torching” for assistance.

Launch Power Boat

   He ordered the big power boat launched and with the crew started on a cruise into the lake to locate, if possible, the cause of the light.  The power boat was headed due west and after running a distance of six or seven miles the light became clearer, but seemed to be but little nearer.  The crew kept on going, however, and at a distance of about ten and twelve miles out, a beautiful panorama of light unfolded before the eyes of the coast guards.

 Captain Preston decided that the flare came from the government lighthouse at Windy Point at Racine.  Being familiar with the Racine lights the keeper was able to identify several of the short lights at Racine, Wis.

Saw Milwaukee Also

   A little further north another set of lights were plainly visible.  Captain Preston knowing the Milwaukee lights well, easily distinguished them and identified them as the Milwaukee lights.  The lights along Juneau Park water front, the illumination of the buildings near the park and the Northwestern Railway station were clearly visible from the Coast Guard boat.  So clearly did the lights stand out that it seemed as though the boat was within a few miles of Milwaukee harbor. 

   Convinced that the phenomenon was a mirage, or a condition due to some peculiarity of the atmosphere, the keeper ordered the boat back to the station.  The lights remained visible for the greater part of the run, and the flare of the Windy Point light house could be seen after the crew reached the station here.


DISTANCE GRAND HAVEN TO MILWAUKEE: OVER 80 MILES (128 KM).

http://www.coastwatch.msu.edu/images/twomichigans2a.gif (http://www.coastwatch.msu.edu/images/twomichigans2a.gif)


Windy Point Lighthouse:

http://upload.wikimedia.org/wikipedia/commons/thumb/5/5f/Wind_Point_Lighthouse_071104_edit2.jpg/800px-Wind_Point_Lighthouse_071104_edit2.jpg (http://upload.wikimedia.org/wikipedia/commons/thumb/5/5f/Wind_Point_Lighthouse_071104_edit2.jpg/800px-Wind_Point_Lighthouse_071104_edit2.jpg)

The lighthouse stands 108 feet (33 m) tall

THE CURVATURE FOR 128 KM IS 321 METERS.

Using the well known formula for the visual obstacle, let us calculate its value:

h = 3 meters BD = 1163 METERS

h = 5 meters BD = 1129 METERS

h = 10 meters BD = 1068 METERS

h = 20 meters BD = 984 METERS

h = 50 meters BD = 827.6 METERS

h = 100 meters BD = 667.6 METERS


No terrestrial refraction formula/looming formula can account for this extraordinary proof that the surface across lake Michigan is flat.

In fact: http://ireland.iol.ie/~geniet/eng/refract.htm# (http://ireland.iol.ie/~geniet/eng/refract.htm#)

If we use h = 50 for the observer, and 140 for the distant object height, we get a negative answer: no way it could be seen over a 128 km distance; while the actual data for the account is h = 5 m, and d = 40 m.


Looming/modified lapse rate:

http://mintaka.sdsu.edu/GF/explain/atmos_refr/altitudes.html (http://mintaka.sdsu.edu/GF/explain/atmos_refr/altitudes.html)

The formula used here does not recognize the change in the range of temperature values, nor do we know if it takes into consideration the very basic formula I posted earlier for the visual obstacle: http://theflatearthsociety.org/forum/index.php?topic=28196.msg674444#msg674444 (http://theflatearthsociety.org/forum/index.php?topic=28196.msg674444#msg674444) - however, it is an excellent place to start and to explore the effect of looming/ducting on the visual target being observed.

Let us use several values, starting with the value of 15 C for that day (Milwaukee/Racine/Holland/Grand Haven) and increasing the value for the target by 1-3 degrees.

For a value of 15 C overall we get of course a negative altitude value of the target.

For a value of 16 C (for the target) we get, again, a negative altitude value for the target (−0.317 degrees of arc) - target is hidden by horizon

For a value of 17 C (for the target) we get: −0.207 degrees of arc, target is hidden by horizon

For a value of 18 C (for the target) we get: −0.098 degrees of arc, target is hidden by horizon


Let us decrease the value to 12 C.

Increasing the value for the target to 15 C degrees, again, we get negative values. This would also correspond to a huge k = 0.613 value.

From the textbook on atmospheric science:

 "So the ray curvature for an arbitrary lapse rate  γ K/m will be

k  = ( 0.034 − γ ) / 0.154

where we take γ to be positive if the temperature decreases with height, and a positive curvature means a ray concave toward the Earth.

Example 1: the Standard Atmosphere:

In the Standard Atmosphere, the lapse rate is 6.5°/km or  γ = 0.0065 K/m. The numerator of the formula above becomes .034 − .0065 = .0275, so the ratio k is about 1/5.6 or 0.179. In other words, the ray curvature is not quite 18% that of the Earth; the radius of curvature of the ray is about 5.6 times the Earth's radius.

Example 2: free convection:

In free convection, the (adiabatic) lapse rate is about 10.6°/km or  γ = 0.0106 K/m. The numerator of the formula above becomes .034 − .0106 = .0234, so the ratio k is about 1/6.6 or 0.152. In other words, the ray curvature is about 15% that of the Earth; the radius of curvature of the ray is about 6.6 times the Earth's radius. This is close to the condition of the atmosphere near the ground in the middle of the day, when most surveying is done; the value calculated is close to the values found in practical survey work."


Moreover, as we have seen, the light from Windy Point was continuously observed, during the approach, and during the return to the station:

The power boat was headed due west and after running a distance of six or seven miles the light became clearer, but seemed to be but little nearer.  The crew kept on going, however, and at a distance of about ten and twelve miles out, a beautiful panorama of light unfolded before the eyes of the coast guards.

The keeper ordered the boat back to the station.  The lights remained visible for the greater part of the run, and the flare of the Windy Point light house could be seen after the crew reached the station here.



Now, the calculation for the most pronounced form of looming: ducting.

However, ducting requires the value for the ray curvature, k, to be greater than or equal to 1.

This amounts to at least a five degree difference in temperature.

With 10C in Grand Haven (or Holland) and 15C in Racine, we get k = 1.182.


For the very same geographical/hydrographical conditions, for the same latitude in question, for cities located on the opposite shores of Lake Michigan, it is absolutely impossible to have a five degree difference, at the very same instant of time - moreover, looming/ducting do not apply to the two cases presented here:

FURTHERMORE, as we have seen, the light from the lighthouse located in Racine was seen all of the time.

For the second case exemplifed here, see below, Mr. Kanis did see the very shape of the buildings: in the case of ducting/looming a very distorted image would appear making it instantly recognizable:

http://upload.wikimedia.org/wikipedia/commons/f/f4/Superopr_mirage_sequence.jpg (http://upload.wikimedia.org/wikipedia/commons/f/f4/Superopr_mirage_sequence.jpg)
http://3sky.de/Div/Luftspieg/Summary.html (http://3sky.de/Div/Luftspieg/Summary.html)
http://finland.fi/public/default.aspx?contentid=160069&contentlan=2&culture=en-US (http://finland.fi/public/default.aspx?contentid=160069&contentlan=2&culture=en-US)




(https://image.ibb.co/irqVco/m11.jpg)
(https://image.ibb.co/mBT0co/m12.jpg)

'As twilight deepened, there were more and more lights.'

Bringing out a pair of binoculars, Kanis said he was able to make out the shape of some buildings.

'With the binoculars we could make out three different communities,' Kanis said.

According to one Coast Guard crewman, it is possible to see city lights across the lake at very specific times.

Currently a Coast Guard crewman stationed in Holland, Todd Reed has worked on the east side of Lake Michigan for 30 years and said he's been able to see lights across the lake at least a dozen times.

The highest building in Milwaukee has a height of 183 meters, the difference from h = 5 meters in altitude being 946 meters, and those residents saw the buildings from THREE DIFFERENT COMMUNITIES, two of which have buildings whose heights measure way under 183 meters.

Therefore, the only way those buildings could be seen, given the 128 km distance, would be if the surface of Lake Michigan is completely flat.

THE TALLEST BUILDING IN RACINE IS THE COUNTY COURTHOUSE, 40 METERS; IT WOULD BE ABSOLUTELY IMPOSSIBLE TO SEE THIS COURTHOUSE FROM 128 KM DISTANCE, FROM HOLLAND.


Title: Re: Earths Curvature and spin effect on long range ballistics.
Post by: Curiosity File on October 04, 2018, 09:44:43 PM
Earthly physically observable phenomena gives way to the necessity of understanding, describing and creating formulas to calculate and give explanation.
 
Surface Ocean Currents 1 2 3 4
   arrow back to previous page       arrow forward to next page
Atmospheric circulation sans earth rotation.
 
If the Earth did not rotate on its axis, the atmosphere would only circulate between the poles and the equator in a simple back-and-forth pattern. Click the image for a larger view.
image bar
 
Coriolis effect on atmospheric circulation.
 
Because the Earth rotates on its axis, circulating air is deflected toward the right in the Northern Hemisphere and toward the left in the Southern Hemisphere. This deflection is called the Coriolis effect. Click the image for a larger view.
image bar

Coastal currents are affected by local winds. Surface ocean currents, which occur on the open ocean, are driven by a complex global wind system. To understand the effects of winds on ocean currents, one first needs to understand the Coriolis force and the Ekman spiral.

Coriolis Effect

If the Earth did not rotate and remained stationary, the atmosphere would circulate between the poles (high pressure areas) and the equator (a low pressure area) in a simple back-and-forth pattern. But because the Earth rotates, circulating air is deflected. Instead of circulating in a straight pattern, the air deflects toward the right in the Northern Hemisphere and toward the left in the Southern Hemisphere, resulting in curved paths. This deflection is called the Coriolis effect. It is named after the French mathematician Gaspard Gustave de Coriolis (1792-1843), who studied the transfer of energy in rotating systems like waterwheels. (Ross, 1995).

https://oceanservice.noaa.gov/education/kits/currents/05currents1.html


    The Coriolis effect describes the pattern of deflection taken by objects not firmly connected to the ground as they travel long distances around and above the Earth. The Coriolis effect is responsible for many large-scale weather patterns.
     
    The key to the Coriolis effect lies in the Earth’s rotation. Specifically, the Earth rotates faster at the Equator than it does at the poles. Earth is wider at the Equator, so to make a rotation in one 24-hour period, equatorial regions race nearly 1,674 kilometers per hour (1,040 miles per hour). Near the poles, the Earth rotates at a sluggish .00008 kph (.00005 mph).
     
    Let’s pretend you’re standing at the Equator and you want to throw a ball to your friend in the middle of North America. If you throw the ball in a straight line, it will appear to land to the right of your friend because he’s moving slower and has not caught up.
     
    Now let’s pretend you’re standing at the North Pole. When you throw the ball to your friend, it will again to appear to land to the right of him. But this time, it’s because he’s moving faster than you are and has moved ahead of the ball.
     
    Everywhere you play global-scale "catch" in the Northern Hemisphere, the ball will deflect to the right.
     
    This apparent deflection is the Coriolis effect. Fluids traveling across large areas, such as air currents, are like the path of the ball. They appear to bend to the right in the Northern Hemisphere. The Coriolis effect behaves the opposite way in the Southern Hemisphere, where currents to bend to the left.
     
    The impact of the Coriolis effect is dependent on velocity—the velocity of the Earth and the velocity of the object or fluid being deflected by the Coriolis effect. The impact of the Coriolis effect is most significant with high speeds or long distances.
     
    Weather Patterns
     
    The development of weather patterns, such as cyclones and trade winds, are examples of the impact of the Coriolis effect.
     
    Cyclones are low-pressure systems that suck air into their center “eye.” In the Northern Hemisphere, fluids from high-pressure systems pass low-pressure systems to their right. As air masses are pulled into cyclones from all directions, they are deflected, and the storm system—a hurricane—seems to rotate counter-clockwise.
     
    In the Southern Hemisphere, currents are deflected to the left. As a result, storm systems seem to rotate clockwise.
     
    Outside storm systems, the impact of the Coriolis effect helps define regular wind patterns around the globe.
     
    As warm air rises near the Equator, for instance, it flows toward the poles. In the Northern Hemisphere, these warm air currents are deflected to the right (east) as they move northward. The currents descend back toward Earth at about 30° north latitude. As the current descends, it gradually moves from the northeast to the southwest, back toward the Equator. The consistently circulating patterns of these air masses are known as trade winds.
     
    Impact on Human Activity
     
    The weather impacting fast-moving objects, such as airplanes and rockets, is influenced by the Coriolis effect. The directions of prevailing winds are largely determined by the Coriolis effect, and pilots must take that into account when charting flight paths over long distances.
     
    Military snipers sometimes have to consider the Coriolis effect. Although the trajectory of bullets is too short to be greatly impacted by the Earth’s rotation, sniper targeting is so precise that a deflection of several centimeters could injure innocent people or damage civilian infrastructure.
     
    The Coriolis Effect on Other Planets
     
    The Earth rotates fairly slowly, compared with other planets. The slow rotation of the Earth means the Coriolis effect is not strong enough to be seen at slow speeds over short distances, such as the draining of water in a bathtub.
     
    Jupiter, on the other hand, has the fastest rotation in the solar system. On Jupiter, the Coriolis effect actually transforms north-south winds into east-west winds, some traveling more than 610 kilometers per hour (380 miles per hour).
     
    The divisions between winds that blow mostly to the east and those that blow mostly to the west create clear horizontal divisions, called belts, among the planet’s clouds. The boundaries between these fast-moving belts are incredibly active storm regions. The 180-year-old Great Red Spot is perhaps the most famous of these storms.
     
    The Coriolis Effect Closer to Home
     
    Despite the popular urban legend, you cannot observe the Coriolis effect by watching a toilet flush or a swimming pool drain. The movement of fluids in these basins is dependent on manufacturer’s design (toilet) or outside forces such as a strong breeze or movement of swimmers (pool).
     
    You can observe the Coriolis effect without access to satellite imagery of hurricanes, however. You could observe the Coriolis effect if you and some friends sat on a rotating merry-go-round and threw or rolled a ball back and forth.
     
    When the merry-go-round is not rotating, rolling the ball back-and-forth is simple and straightforward. While the merry-go-round is rotating, however, the ball won’t make to your friend sitting across from you without significant force. Rolled with regular effort, the ball appears to curve, or deflect, to the right.
     
    Actually, the ball is traveling in a straight line. Another friend, standing on the ground near the merry-go-round, will be able to tell you this. You and your friends on the merry-go-round are moving out of the path of the ball while it is in the air.
    Coriolis effect
    Storms in the north swing counter-clockwise: the Coriolis effect.
    Storms in the south swing with the clock, and winds tend to pass to the left!

    Photograph courtesy NASA, Jeff Schmaltz, MODIS Land Rapid Response Team at NASA GSFC

    Coriolis Force
    The invisible force that appears to deflect the wind is the Coriolis force. The Coriolis force applies to movement on rotating objects. It is determined by the mass of the object and the object's rate of rotation. The Coriolis force is perpendicular to the object's axis. The Earth spins on its axis from west to east. The Coriolis force, therefore, acts in a north-south direction. The Coriolis force is zero at the Equator.

    Though the Coriolis force is useful in mathematical equations, there is actually no physical force involved. Instead, it is just the ground moving at a different speed than an object in the air.
    Polar Power
    The Coriolis force is strongest near the poles, and absent at the Equator. Cyclones need the Coriolis force in order to circulate. For this reasons, hurricanes almost never occur in equatorial regions, and never cross the Equator itself.

Articles & Profiles

    Professor Seligman: Coriolis Effects
    Hayden Planetarium: Neil DeGrasse Tyson—The Coriolis Force
    NOAA Ocean Service Education: Surface Ocean Currents
    NOAA SciJinks: What is the Coriolis Effect?

https://www.nationalgeographic.org/encyclopedia/coriolis-effect/
Title: Re: Earths Curvature and spin effect on long range ballistics.
Post by: Curiosity File on October 04, 2018, 09:57:11 PM
The formulas related to Coriolis acceleration and Coriolis force can be found here:

Coriolis force - Wikipedia

(UNABLE TO COPY THE FORMULA. GO TO THE LINK)
The big omega (Ω

) is the angular velocity of the rotation frame of reference, which is the angular velocity of the earth about its polar axis. The velocity of the object that experiences the effect is v. Its mass is m.

It’s that cross-product vector operator that helps us figure out what would happen at the poles and at the equator. The cross product is zero when the two vectors are in the same direction (including directly opposite). It is largest when the angle between them is 90°.

Ω

is parallel to the polar axis. So if you are at the equator and moving north or south (which is exactly parallel to the polar axis), then the Coriolis acceleration and force are zero. If you are traveling east or west, then the effect is present, but its direction of action is vertical. in this case, the Coriolis force pushes straight up or straight down. It changes your weight ever so slightly. So the Coriolis effect exists at the equator, it just doesn’t have an effect that we notice.

At the poles, any velocity along the surface of the earth is at 90° to the polar axis and so the Coriolis effects pushes sideways for all those cases. If you jumped vertically at the pole, there would be no Coriolis force since your velocity would be parallel to the axis of rotation and the result of the cross product would be zero.

If you jumped vertically at the equator, then that would produce a Coriolis force. In some sense, rockets launched at the equator take advantage of that Coriolis force to help them go into orbit in an easterly direction.
12.9k Views · View Upvoters

https://www.quora.com/Why-is-the-Coriolis-effect-zero-at-the-equator


https://www.youtube.com/watch?time_continue=1&v=aeY9tY9vKgs
Title: Re: Earths Curvature and spin effect on long range ballistics.
Post by: Humble B on October 04, 2018, 10:01:00 PM
Within science this is called: "Evidence for a spinning globe".

You will be disappointed again.

The FE equation with a fixed g is the WRONG formula.

g is a variable.

As such, we can immediately obtain the correct value for the entire calculation.

Within science this is called "Fraud"; changing a physical constant into a variable to make the outcome of a calculation fit your philosophical agenda.

Spinning globe you say...

But there is no curvature whatsoever across lake Michigan.

 
Grand Haven Daily Tribune   April 3, 1925

COAST GUARDS SEE MILWAUKEE LIGHTS GLEAM.....

That this 93 years afterwards still is remembered as a rare and unexplainable occasion proves there is a lot of curvature across lake Michigan, otherwise such observations would be that commonplace that nobody would bother talking about it, and we in Holland Europe would be accustomed to see the English coastline and lighthouses from our dunes.

Title: Re: Earths Curvature and spin effect on long range ballistics.
Post by: Mysfit on October 04, 2018, 10:05:40 PM
Given that there is a few inches difference on the targets, I would consider the effect a waste of time to account for at shorter distances.

Unless the sniper is 100% on all other hard-to-predict variables, a few inches can be the difference between a kill shot and a miss.

Arguing that snipers do not use Coriolis does because it is a 'waste of time' does not help the Round Earth argument that snipers account for Coriolis when sniping. That significantly weakens the argument that this effect actually exists.

I think you misunderstood my point, I was specifying that the effect is negligible enough at ranges of 100 to 1000 yards that it would still guarantee an effective hit.
I acknowledge that below 1000 yards, it would likely not be worth mentioning in the manual. The snipers are likely being asked to calculate the most important factors. In war, an injury is much more useful than a kill, it affects morale, takes time and resources to treat and still, effectively, takes them out of the fight.

Given that the fellow in the video is trying to sell a more accurate system, it would be in his best interest to focus on the Coriolis minutia. Especially if his market is for folks who shoot more than 1000 yards or are fascinated with every bit of gun they can get.
I’m glad you’re not arguing that the guy in the video was being deceptive or the results were skewed for some reason. I wouldn’t be able to argue against it, but it would make any further attempts at truth seeking pointless.

Also, I don’t care about Coriolis proving a round world. I know it exists and am happy for your model to be an interesting thought experiment. I also think I can bring it into the fold of a non-spinning disc without relying on ether/aether (working on it for a diff post).
Title: Re: Earths Curvature and spin effect on long range ballistics.
Post by: Dr David Thork on October 04, 2018, 11:18:28 PM
I think you can step away from the maths for a short while.


Let's say you fire a bullet North from the equator. RET states that bullet will miss its target and go a few inches to the right as the bullet went straight and the earth moved left.

Now consider an aircraft flying North. The aircraft goes North and the earth moves left. In fact the earth moves so far left, it wouldn't be possible to land at the airfield to the North as the earth moves away faster than the aircraft can fly.

So, you seem to want it both ways. You insist a bullet must be affected by Coriolis and that an aircraft isn't. Which is it? They both fly through the air (they can both be doing comparable speeds), the aircraft has a longer time in the air and travels further so should be more affected. But the aircraft has no affect at all ... and yet the bullet somehow does. This is picking and choosing. The thing you know to be true (an aircraft isn't affected by Coriolis as it flies North) you ignore as it doesn't fit your theory ... and the thing you haven't checked (firing a bullet North accurately) you take as read because Hollywood makes a thing about snipers being so good they even take into account the spin of the earth when they fire.

I'd like to know why Coriolis affects bullets and why aircraft are immune to its power, please? FET has this covered ... no Coriolis. Neither affected. How to you end up with selective Coriolis?
Title: Re: Earths Curvature and spin effect on long range ballistics.
Post by: Curiosity File on October 04, 2018, 11:25:24 PM
Because inertial forces like the Coriolis force and the centrifugal force do not exist.

Completely wrong.

The author of this statement has no knowledge of the original set of J.C. Maxwell's equations, which do include the Coriolis term/vorticity.

Dr. Frederick Tombe explains:

Not at all completely wrong.

The authors and papers you are referring to do not dispute the fact that the Coriolis force is an inertial force, and that the path of a bullet over the surface of the earth is straight, not curved.

My point was that if the earth is not spinning, and the bullets trajectory is curved, then this curve can not be the result of an "inertial force" as you claim, and the curved trajectory of the bullet above a stationary earth has nothing to do with the Coriolis effect.

Even if science would change their mind about the fictitious status of inertial forces, you (those who claim the physical the earth is not spinning) still need another force than an inertial one to prove your case, because above a stationary surface a curved trajectory has nothing to do with the Coriolis effect.

I'm to understand that without Coriolis inertial force created by the earth moving causing the bullet to go right or left of the target, we have a stationary earth? That leave us to believe there is a force pushing the bullet off target?
I'm to understand this force is created by the mas of stars moon and sun along with matter Moving in one direction?
What I don't understand is how the bullet is deflected the opposite direction what we see in the sky no mater which hemisphere you're on?
Title: Re: Earths Curvature and spin effect on long range ballistics.
Post by: Curiosity File on October 05, 2018, 01:00:22 AM
I think you can step away from the maths for a short while.


Let's say you fire a bullet North from the equator. RET states that bullet will miss its target and go a few inches to the right as the bullet went straight and the earth moved left.

Now consider an aircraft flying North. The aircraft goes North and the earth moves left. In fact the earth moves so far left, it wouldn't be possible to land at the airfield to the North as the earth moves away faster than the aircraft can fly.

So, you seem to want it both ways. You insist a bullet must be affected by Coriolis and that an aircraft isn't. Which is it? They both fly through the air (they can both be doing comparable speeds), the aircraft has a longer time in the air and travels further so should be more affected. But the aircraft has no affect at all ... and yet the bullet somehow does. This is picking and choosing. The thing you know to be true (an aircraft isn't affected by Coriolis as it flies North) you ignore as it doesn't fit your theory ... and the thing you haven't checked (firing a bullet North accurately) you take as read because Hollywood makes a thing about snipers being so good they even take into account the spin of the earth when they fire.

I'd like to know why Coriolis affects bullets and why aircraft are immune to its power, please? FET has this covered ... no Coriolis. Neither affected. How to you end up with selective Coriolis?
Your facts are incorrect.
#1 on average bullets fly twice the speed of commercial jets.
#2 Lateral Inertial forces on the bullet are greater at the equator than the lateral movement of the target to the north, is why it hits to the right of target.
But shooting from north of the equator to the equator the lateral inertial force of the bullet is slower than the target is moving to the east. Again causing it to hit to the right.
Aircraft are affected less due to mass and many other factors including propulsion, ability to adjust direction of path while in flight and wings that cause them to be highly influence by air pressure.

https://davidson.weizmann.ac.il/en/online/askexpert/physics/motion-planes-influenced-rotation-earth



Planes and bullets are both affected by Coriolis effects. The bullet will be more affected than the plane, because it's velocity is higher, but that's only the tiniest part of the story.

Bullets are unguided projectiles. They go where the physics says they go. Any effects due to Coriolis are unabated. Also, bullets which are fired at distances where Coriolis really start to matter are also trying to hit a very small target. Small effects are very noticeable.

Planes are controlled. There is a guidance system (such as a pilot) which is trying to keep it on course. This is important because there are forces that will deflect the plane off course which are orders of magnitude more powerful than the Coreolis effect. If there was not active correction, those other forces would virtually guarantee that we never arrived at our destination.

Because there's a guidance system in play, we rapidly trim the aircraft to counteract the Coreolis effect as a side effect of trying to trim the aircraft for all of the much bigger forces at play. There may be a slight deflection of the rudder associated with the Coreolis effect, but you would be hard pressed to identify it amidst all the other forces.

https://physics.stackexchange.com/questions/171048/coriolis-force-on-bullet-vs-airplane

   
Title: Re: Earths Curvature and spin effect on long range ballistics.
Post by: Tom Bishop on October 05, 2018, 01:24:33 AM
Quote
Your facts are incorrect.
#1 on average bullets fly twice the speed of commercial jets.
#2 Lateral Inertial forces on the bullet are greater at the equator than the lateral movement of the target to the north, is why it hits to the right of target.
But shooting from north of the equator to the equator the lateral inertial force of the bullet is slower than the target is moving to the east. Again causing it to hit to the right.
Aircraft are affected less due to mass and many other factors including propulsion, ability to adjust direction of path while in flight and wings that cause them to be highly influence by air pressure.

https://davidson.weizmann.ac.il/en/online/askexpert/physics/motion-planes-influenced-rotation-earth

Planes and bullets are both affected by Coriolis effects. The bullet will be more affected than the plane, because it's velocity is higher, but that's only the tiniest part of the story.

Bullets are unguided projectiles. They go where the physics says they go. Any effects due to Coriolis are unabated. Also, bullets which are fired at distances where Coriolis really start to matter are also trying to hit a very small target. Small effects are very noticeable.

Planes are controlled. There is a guidance system (such as a pilot) which is trying to keep it on course. This is important because there are forces that will deflect the plane off course which are orders of magnitude more powerful than the Coreolis effect. If there was not active correction, those other forces would virtually guarantee that we never arrived at our destination.

Because there's a guidance system in play, we rapidly trim the aircraft to counteract the Coreolis effect as a side effect of trying to trim the aircraft for all of the much bigger forces at play. There may be a slight deflection of the rudder associated with the Coreolis effect, but you would be hard pressed to identify it amidst all the other forces.

https://physics.stackexchange.com/questions/171048/coriolis-force-on-bullet-vs-airplane

This is the most ridiculous understanding of physics I have encountered. Why should it matter whether the bullet or airplane is moving faster or slower, as to be affected by the Coriolis Effect? Is there some kind of speed detector where this effect turns on?
Title: Re: Earths Curvature and spin effect on long range ballistics.
Post by: stack on October 05, 2018, 01:32:32 AM
Where are those Tycho Bahe experiments you claimed proved no rotation of the earth?
Title: Re: Earths Curvature and spin effect on long range ballistics.
Post by: Curiosity File on October 05, 2018, 01:40:29 AM
Quote
Your facts are incorrect.
#1 on average bullets fly twice the speed of commercial jets.
#2 Lateral Inertial forces on the bullet are greater at the equator than the lateral movement of the target to the north, is why it hits to the right of target.
But shooting from north of the equator to the equator the lateral inertial force of the bullet is slower than the target is moving to the east. Again causing it to hit to the right.
Aircraft are affected less due to mass and many other factors including propulsion, ability to adjust direction of path while in flight and wings that cause them to be highly influence by air pressure.

https://davidson.weizmann.ac.il/en/online/askexpert/physics/motion-planes-influenced-rotation-earth

Planes and bullets are both affected by Coriolis effects. The bullet will be more affected than the plane, because it's velocity is higher, but that's only the tiniest part of the story.

Bullets are unguided projectiles. They go where the physics says they go. Any effects due to Coriolis are unabated. Also, bullets which are fired at distances where Coriolis really start to matter are also trying to hit a very small target. Small effects are very noticeable.

Planes are controlled. There is a guidance system (such as a pilot) which is trying to keep it on course. This is important because there are forces that will deflect the plane off course which are orders of magnitude more powerful than the Coreolis effect. If there was not active correction, those other forces would virtually guarantee that we never arrived at our destination.

Because there's a guidance system in play, we rapidly trim the aircraft to counteract the Coreolis effect as a side effect of trying to trim the aircraft for all of the much bigger forces at play. There may be a slight deflection of the rudder associated with the Coreolis effect, but you would be hard pressed to identify it amidst all the other forces.

https://physics.stackexchange.com/questions/171048/coriolis-force-on-bullet-vs-airplane

This is the most ridiculous understanding of physics I have encountered. Why should it matter whether the bullet or airplane is moving faster or slower, as to be affected by the Coriolis Effect? Is there some kind of speed detector where this effect turns on?
What's there to not understand about how hugely everything is affected differently by different velocities.

Nothing turns on. It's explained in articles in the links I shared to the Davidson institution of Science and Education.
It's pretty bold to of you Tom to site that as a ridiculous understanding of physics. And by the way that sounded like a personal attack on me or the DISE, or both. I thought this was a site that debated and discuses information?
But anyway, more than having to do with speed and distance I think is has to do with the fact that comparing a plane to a bullet is like comparing a ball to a sheet of paper.
But here's a little inexpensive experiment you can do Tom.
Make a paper airplane, take a marble, throw them and note how different they act.
 
   

 
Title: Re: Earths Curvature and spin effect on long range ballistics.
Post by: Curiosity File on October 05, 2018, 04:07:08 AM
Given that there is a few inches difference on the targets, I would consider the effect a waste of time to account for at shorter distances.

Unless the sniper is 100% on all other hard-to-predict variables, a few inches can be the difference between a kill shot and a miss.

Arguing that snipers do not use Coriolis does because it is a 'waste of time' does not help the Round Earth argument that snipers account for Coriolis when sniping. That significantly weakens the argument that this effect actually exists.

I think you misunderstood my point, I was specifying that the effect is negligible enough at ranges of 100 to 1000 yards that it would still guarantee an effective hit.
I acknowledge that below 1000 yards, it would likely not be worth mentioning in the manual. The snipers are likely being asked to calculate the most important factors. In war, an injury is much more useful than a kill, it affects morale, takes time and resources to treat and still, effectively, takes them out of the fight.

Given that the fellow in the video is trying to sell a more accurate system, it would be in his best interest to focus on the Coriolis minutia. Especially if his market is for folks who shoot more than 1000 yards or are fascinated with every bit of gun they can get.
I’m glad you’re not arguing that the guy in the video was being deceptive or the results were skewed for some reason. I wouldn’t be able to argue against it, but it would make any further attempts at truth seeking pointless.

Also, I don’t care about Coriolis proving a round world. I know it exists and am happy for your model to be an interesting thought experiment. I also think I can bring it into the fold of a non-spinning disc without relying on ether/aether (working on it for a diff post).

A couple things I would like point.
#1 one main component of their job is the "kill" shot. they are assassins.
# I also think Tom misunderstood, but I'd still like to point out that using the minute measurable deflection at such short distances absolutely strengthens the argument of this effects existence..     
Title: Re: Earths Curvature and spin effect on long range ballistics.
Post by: sandokhan on October 05, 2018, 05:23:11 AM
Within science this is called "Fraud"; changing a physical constant into a variable to make the outcome of a calculation fit your philosophical agenda.

Science is based on experiment.

It takes a single counterexample to invalidate a hypothesis.

NIPHER EFFECT

“Dr. Francis Nipher, Professor of physics, Washington University, St. Louis, Missouri, did some of the pioneering electrogravitics work at Washington University in St. Louis back around the turn of the last century. He applied high voltage to lead balls, lead spheres and hollow metal boxes and compared the repulsive effect induced in small test spheres hung vertically near them, similar to the original Cavendish experiments but with high voltage. Dr. Nipher went to great lengths to insert protective, grounded screens of glass between the solid lead spheres and the suspended balls to rule out electrostatic effects.”

Before connecting any form of electric current to the modified Cavendish apparatus, Prof.  Nipher took special precaution to carefully screen the moving element from any electrostatic or electromagnetic effects. His apparatus briefly consists of two large lead spheres ten inches in diameter, resting upon heavy sheets of hard rubber. Two small lead balls, each one inch in diameter, were now suspended from two silk threads, stationed at the sides of the two large lead spheres, from which they were separated by a little distance. Moreover, the suspended balls were insulated elaborately from the large spheres by enclosing them first airtight in a long wooden box, which was also covered with tinned iron sheets as well as cardboard sheets. There was, furthermore, a metal shield between the box and the large metal spheres. The large metal lead spheres now exerted a certain gravitational force upon the suspended small lead balls … and the small lead balls were slightly moved over towards the large spheres.

In further experiments Prof.  Nipher decided to check his results. To do this he replaced the large solid lead spheres with two metal boxes, each filled with loose cotton batting. These hollow boxes (having practically no mass) rested upon insulators. They were separated from the protective screen by sheets of glass and were grounded to it by heavy copper wires. The metal boxes were then charged in every way that the solid lead spheres had been, but not the slightest change in the position of the lead balls could be detected. This would seem to prove conclusively that the "repulsion" and "gravitational nullification" effects that he had produced when the solid balls were electrically charged were genuine and based undoubtedly on a true inter-atomic electrical reaction, and not upon any form of electrostatic or electromagnetic effects between the large and small masses. If they had been, the metal boxes, with no mass, would have served as well as the solid balls.


The relationship between gravitation and the electric field was first observed experimentally by Dr. Francis Nipher. Nipher's conclusion was that sheilded electrostatic fields directly influence the action of gravitation. He further concluded that gravitation and electrical fields are absolutely linked.

http://www.rexresearch.com/nipher/nipher1.htm

New Evidence of a Relation Between Gravitation & Electrical Action (1920)
Gravitational Repulsion (1916)
Gravitation & Electrical Action (1916)
Can Electricity Reverse the Effect of Gravity? (1918)

The relationship between gravitation and the electric field was first observed experimentally by Dr. Francis Nipher. Dr. Francis Nipher conducted extensive experiments during 1918, on a modified Cavendish experiment. He reproduced the classical arrangements for the experiment, where gravitational attraction could be measured between free-swinging masses, and a large fixed central mass. Dr. Nipher modified the Cavendish experiment by applying a large electrical field to the large central mass, which was sheilded inside a Faraday cage. When electrostatic charge was applied to the large fixed mass, the free-swinging masses exhibited a reduced attraction to the central mass, when the central mass was only slightly charged. As the electric field strength was increased, there arose a voltage threshold which resulted in no attraction at all between the fixed mass and the free-swinging masses. Increasing the potential applied to the central mass beyond that threshold, resulted in the free-swinging masses being repelled (!) from the fixed central mass. Nipher's conclusion was that sheilded electrostatic fields directly influence the action of gravitation. He further concluded that gravitation and electrical fields are absolutely linked.


"These results seem to indicate clearly that gravitational attraction between masses of matter depends upon electrical potential due to electrical charges upon them."

Every working day of the following college year has been devoted to testing the validity of the above statement. No results in conflict with it have been obtained. Not only has gravitational attraction been diminished by electrification of the attracting bodies when direct electrical action has been wholly cut off by a metal shield, but it has been made negative. It has been converted into a repulsion. This result has been obtained many times throughout the year. On one occasion during the latter part of the year, this repulsion was made somewhat more than twice as great as normal attraction."

Increasing the potential applied to the central mass beyond that threshold, resulted in the free-swinging masses being repelled (!) from the fixed central mass. Nipher's conclusion was that sheilded electrostatic fields directly influence the action of gravitation. He further concluded that gravitation and electrical fields are absolutely linked.

Dr. Francis Nipher one of the most distinguished physicists of the United States:

http://www.accessgenealogy.com/missouri/biography-of-francis-eugene-nipher-ll-d.htm


Modern science is a FRAUD: it is ignoring the definite proofs obtained by Professor Nipher, g is a variable.


BIEFELD-BROWN EFFECT

Dr. Paul Biefeld used to do Einstein's homework while they were studying at the  Polytechnic University in Zurich in 1900.

Dr. Paul Biefeld was a classmate of A. Einstein.

 “Yes,” Biefeld told the Denison campus newspaper, “when Einstein would forget to go to a class, he would come and borrow my notes to get caught up on what he had missed."

Experimental proof that the vacuum consists of ether.

https://youtu.be/CGN65lse5yE

(vacuum test performed by Gravitec, increasing the voltage from 15kv to 18 kv, clear movement/thrust of the capacitor can be seen; near the end the power is switched off, and then turned on again, and we can the visible thrust of the capacitor for a second time)

The first video supplied by Gravitec in 2003:

https://www.theflatearthsociety.org/forum/index.php?topic=30499.msg1852363#msg1852363 (vacuum test #1, Biefeld-Brown effect part I, contains the experiments performed by T. Brown in oil)

https://youtu.be/ZE7Go7ptBRY

One of the best videos which exemplifies the Biefeld-Brown effect in vacuum:

https://web.archive.org/web/20050216062907/http://www-personal.umich.edu/~reginald/liftvac.html


http://lifters.online.fr/lifters/ascvacuum/index.htm (includes all necessary technical information and the video itself)


At the pressure of 1.72 x 10^-6 Torr ( High Vacuum conditions ), the apparatus rotates when the High Voltage is increased from 0 to +45 KV.


Dr. Takaaki Musha
Advanced Space Propulsion Investigation Committee (ASPIC)
Research Engineer on Naval Systems, Technical Research & Development Institute
Honda R&D Institute, Biefeld-Brown effect experiments

http://jnaudin.free.fr/lifters/musha/Musha.pdf

Explanation of dynamical Biefeld-Brown Effect from the standpoint of ZPF field

In 1956, T.T. Brown presented a discovery known as the Biefeld-Bown effect (abbreviated B-B effect) that a sufficiently charged capacitor with dielectrics exhibited unidirectional thrust in the direction of the positive plate.

From the 1st of February until the 1st of March in 1996, the research group of the HONDA R&D Institute conducted experiments to verify the B-B effect with an improved experimental device which rejected the influence of corona discharges and electric wind around the capacitor by setting the capacitor in the insulator oil contained within a metallic vessel . . . The experimental results measured by the Honda research group are shown . . .

. . . The theoretical analysis result suggests that the impulsive electric field applied to the dielectric material may produce a sufficient artificial gravity to attain velocities comparable to chemical rockets.


https://web.archive.org/web/20120710005059/http://www.ovaltech.ca/pdfss/Theoretical_Explanation_of_the_Biefield-Brown_Effect.pdf

Experiments carried out at the HONDA R&D Institute confirm that the Biefeld-Brown effect is real.

Direct experimental proof that the ether propagates through vacuum, causing the antigravitational Biefeld-Brown effect.

No gas left in the experiments: they were performed in vacuum.

Calculations indicate that ionic wind is at least three orders of magnitude too small to explain the magnitude of the observed force on the capacitor (in open air experiments).

Also the experiments carried out at the Honda R&D eliminated the possibility of ionic winds.


That this 93 years afterwards still is remembered as a rare and unexplainable occasion proves there is a lot of curvature across lake Michigan, otherwise such observations would be that commonplace that nobody would bother talking about it, and we in Holland Europe would be accustomed to see the English coastline and lighthouses from our dunes.

You still don't get it.

It is IMPOSSIBLE to see a 40 meter tall building from a distance of 128 km on a spherical earth: no looming/ducting formulas will help you.

The County Courthouse was seen from a distance of 128 km: the observer was located right on the eastern shoreline of lake Michigan.

The lighthouse (height of 33 meters) was seen from the deck of the ship (3-5 meters in height) which was located right next to the eastern shoreline of lake Michigan, over a distance of 128 km.

The curvature measures 321 meters, the visual obstacle measures 1130 meters.

It takes a single counterexample to invalidate a hypothesis.

It is a total FRAUD on your part to go on believing in a "speeding globe" when the proof is very clear: there is no curvature whatsoever across lake Michigan.



Holland you say...

The English Channel: 34 km distance from Cap Gris Nez to Dover, a curvature of some 22.4 meters on a round earth.

(https://image.ibb.co/hPHJxo/dover1.jpg)
(https://image.ibb.co/e1Daco/dover2.jpg)

The original webpages, as they were posted on flickr.com

The photographers located between Cap Blanc Nez and Cap Gris Nez: we will ascend to 30 meters.

(https://image.ibb.co/kf7qA8/doverbest2.jpg)

And now the photograph itself: no curvature whatsoever, all the way to the other shoreline, the Dover cliffs seen in their entirety (on a round earth, from 30 meters, we could not see anything under 16.5 meters from the other side), the ships are not part of an ascending/descending slope, no midpoint curvature of 22.4 meters:

(https://image.ibb.co/jBn7q8/doverbest.jpg)

Another photograph taken right on the beach of Cap Gris Nez: no curvature over a distance of 34 km:

(http://www.expedition360.com/journal/white_cliffs.jpg)

Dover cliffs:

(https://farm5.staticflickr.com/4070/4521816996_2971e62065.jpg)
(https://farm2.static.flickr.com/1051/4726849923_389dba2176.jpg)

No curvature whatsoever across the English Channel: you believe in a TOTAL FRAUD, which manifests itself as cognitive dissonance, the inability to accept reality.
Title: Re: Earths Curvature and spin effect on long range ballistics.
Post by: sandokhan on October 05, 2018, 05:42:52 AM
I'd like to know why Coriolis affects bullets and why aircraft are immune to its power, please? FET has this covered ... no Coriolis. Neither affected. How to you end up with selective Coriolis?

The Coriolis force exerted by the ether drift is real: it was measured by Michelson and Gale in 1925.

The selectiveness is due to the fact that the engines of the airplane will be subjected to the Schauberger effect, in addition to the DePalma effect.

https://www.theflatearthsociety.org/forum/index.php?topic=30499.msg2044376#msg2044376 (Schauberger effect for jet engines, it was Viktor Schauberger who invented the jet engine)


Spinning globe...

Let's put your word to the test.

If the Earth were to orbit the Sun at some 30km/s, the GPS satellites would immediately register the ORBITAL SAGNAC EFFECT, which they do not.

In fact, the GPS on your phone works even because the orbital Sagnac is missing, otherwise the precision would be degraded in an instant of time by hundreds of meters, if not kilometers.


http://qem.ee.nthu.edu.tw/f1b.pdf

This is an IOP article.

The author recognizes the earth's orbital Sagnac is missing whereas the earth's rotational Sagnac is not.

He uses GPS and a link between Japan and the US to prove this.

In GPS the actual magnitude of the Sagnac correction
due to earth’s rotation depends on the positions of
satellites and receiver and a typical value is 30 m, as the
propagation time is about 0.1s and the linear speed due
to earth’s rotation is about 464 m/s at the equator. The
GPS provides an accuracy of about 10 m or better in positioning.
Thus the precision of GPS will be degraded significantly,
if the Sagnac correction due to earth’s rotation
is not taken into account. On the other hand, the orbital
motion of the earth around the sun has a linear speed of
about 30 km/s which is about 100 times that of earth’s
rotation. Thus the present high-precision GPS would be
entirely impossible if the omitted correction due to orbital
motion is really necessary.


In an intercontinental microwave link between Japan and
the USA via a geostationary satellite as relay, the influence
of earth’s rotation is also demonstrated in a high-precision
time comparison between the atomic clocks at two remote
ground stations.
In this transpacific-link experiment, a synchronization
error of as large as about 0.3 µs was observed unexpectedly.


Meanwhile, as in GPS, no effects of earth’s orbital motion
are reported in these links, although they would be
easier to observe if they are in existence. Thereby, it is evident
that the wave propagation in GPS or the intercontinental
microwave link depends on the earth’s rotation, but
is entirely independent of earth’s orbital motion around
the sun or whatever. As a consequence, the propagation
mechanism in GPS or intercontinental link can be viewed
as classical in conjunction with an ECI frame, rather than
the ECEF or any other frame, being selected as the unique
propagation frame. In other words, the wave in GPS or the
intercontinental microwave link can be viewed as propagating
via a classical medium stationary in a geocentric
inertial frame.


These are the calculations performed by some of the best mathematicians in the world for the LISA project.

(https://image.ibb.co/kpLCon/ether2.jpg)

Algebraic approach to time-delay data analysis: orbiting case
K Rajesh Nayak and J-Y Vinet

https://www.cosmos.esa.int/documents/946106/1027345/TDI_FOR_.PDF/2bb32fba-1b8a-438d-9e95-bc40c32debbe

This is an IOP article, published by the prestigious journal Classic and Quantum Gravity:

http://iopscience.iop.org/article/10.1088/0264-9381/22/10/040/meta

"In this work, we estimate the effects due to the Sagnac phase by taking the realistic model for LISA orbital motion."

"Earlier results assume a simple module in which LISA rotates only about its own axis!!

In reality the motion of LISA is much more complex and our study shows that the main term for Sagnac effect comes from orbital motion."

Conclusions:

The contribution from the Sagnac effect is much larger than earlier predicted.

Full calculations comparing the rotational Sagnac with the orbital Sagnac lead to the final result:

(https://image.ibb.co/iMSdB7/lisa3.jpg)

The original arm length for LISA: 5,000,000 km (L)

Earth - Sun radius: 150,000,000 km (R)

ORBITAL SAGNAC/ROTATIONAL SAGNAC =~ R/L = 30


This effect is totally missing from the GPS satellites!

The Earth does not orbit the Sun at all.


(https://image.ibb.co/iSbPJn/lis1.jpg)
(https://image.ibb.co/b2YMyn/lis2.jpg)
(https://image.ibb.co/cStX4S/lis3.jpg)
(https://image.ibb.co/mpRKjS/lisa5.jpg)


The solar gravitational effect upon the GPS satellites is also missing:

https://www.theflatearthsociety.org/forum/index.php?topic=30499.msg1846706#msg1846706


This means that the hypotheses of the RUDERFER EXPERIMENT are fulfilled.

Martin Ruderfer experiment performed in 1960:

Ruderfer, Martin (1960) “First-Order Ether Drift
Experiment Using the Mössbauer Radiation,”
Physical Review Letters, Vol. 5, No. 3, Sept. 1, pp
191-192

Ruderfer, Martin (1961) “Errata—First-Order Ether
Drift Experiment Using the Mössbauer Radiation,”
Physical Review Letters, Vol. 7, No. 9, Nov. 1, p 361


in 1961, M. Ruderfer proved mathematically and experimentally, using the spinning Mossbauer effect, the FIRST NULL RESULT in ether drift theory.

"What students are not told is that the Turner & Hill experiment is a garbled version of a 1960 investigation by Ruderfer, who was seeking to discover fluctuations in gamma ray frequency which might indicate motion of an electromagnetic medium across the plane of the spinning disk, causing cyclic Doppler-type changes in the transit times of the gamma rays crossing that disk. Initially Ruderfer put it out that his results were negative for ether drift, but 14 months later he published an errata which stated that mathematical analysis had shown that if an ether wind were blowing across the plane of the spinning disk, one would expect that Doppler fluctuations in the frequency of the gamma radiation detected at the centre of the disk would be compensated by equal and opposite fluctuations in the emitted frequency of the gamma rays, caused by the effect of variations in the ether speed of the source.

What Ruderfer's experiment had stumbled on was that there could be a static electromagnetic medium at rest with respect to the rest of the universe. And it could be that any motion with respect to that medium affects the gamma ray source, and the central Mossbauer detector, by slowing down the rate of process of each by half the square of the ratio of each one's absolute ether speed to the absolute speed of propagation of light. If such were the case, it would follow (as a mathematical necessity) that irrespective of the direction and speed of ether drift of the lab, the central detector of the spinning disk would always observe a steady slowing of the gamma radiation frequency by half the square of the ratio of the spin speed of the source to the out-and-return speed of light, as measured by the detector in a reference frame which is non-rotating with respect to the fixed stars.

Ruderfer's experiment and his errata were of great significance in the history of modern physics because of their psychological impact on the ether deniers. Previously, the Michelson & Morley ether drift experiment had been successfully portrayed as 'negative' rather than 'null' because the proposed compensating factor, Fitzgerald contraction, was a theoretical construct. However, in the case of the Ruderfer experiment, the ether deniers were shocked to find that the experiment provided proof of the existence of the compensating factor in the observed frequency reduction, making it indubitably a null ether drift experiment.

Since the motion-induced frequency reduction of the gamma ray source is by a steady 'half the square of the ratio of the disk spin speed to the speed of propagation of the gamma rays', and since this is exactly the amount required to give the same result, irrespective of whether the disk is at ether rest, or is orientated edgewise (or at right angles) to a hypothetical ether drift, this constituted prima facie evidence for something for which the ether deniers have a particular fear and loathing - 'laws of nature which conspire to conceal the effect of ether drift'."

https://web.archive.org/web/20070315063351/http://egtphysics.net/Index.htm (select the Ether Drift article option)

http://www.tuks.nl/pdf/Reference_Material/Ronald_Hatch/Hatch-Clock_Behavior_and_theSearch_for_an_Underlying_Mechanism_for_Relativistic_Phenomena_2002.pdf


Analysis of the spinning Mossbauer experiments is a natural step toward analysis of the
slightly more complex and much larger-scale Global Positioning System (GPS). This
system constitutes a large scale near-equivalent to the spinning Mossbauer experiments.
The transit time between the satellite and ground-based receivers is routinely measured.
In addition, the atomic clocks on the satellite are carefully monitored; and high precision
corrections are provided as part of the information transmitted from the satellites.
Because the satellites and the receivers rotate at different rates (unlike the Mossbauer
experiments), a correction for the motion of the receiver during the transit time is
required. This correction is generally referred to as a Sagnac correction, since it adjusts
for anisotropy of the speed of light as far as the receiver is concerned. Why is there no
requirement for a Sagnac correction due to the earth’s orbital motion? Like the transit
time in the spinning Mossbauer experiments, any such effect would be completely
canceled by the orbital-velocity effect on the satellite clocks.


Specifically, there is substantial independent experimental evidence that clock speed always affects the clock frequency and, as the GPS system shows, the spin velocity of the earth clearly affects the clock rate. This being the case, the null result of the rotating Mössbauer experiments actually implies that an ether drift must exist or else the clock effect would not be canceled and a null result would not be present.

A GPS satellite orbiting the Earth, while at the same time the entire system is orbiting the Sun, IS A LARGE SCALE SPINNING MOSSBAUER EXPERIMENT.


Given the very fact that these GPS satellites DO NOT record the orbital Sagnac effect, means that THE HYPOTHESES OF THE RUDERFER EXPERIMENT ARE FULFILLED.

Why is there no requirement for a Sagnac correction due to the earth’s orbital motion? Like the transit time in the spinning Mossbauer experiments, any such effect would be completely canceled by the orbital-velocity effect on the satellite clocks.

However, indirectly, the counteracting effects of the transit time and clock slowing induced biases indicate that an ether drift is present. This is because there is independent evidence that clocks are slowed as a result of their speed. Thus, ether drift must exist or else the clock slowing effect would be observed.

In fact, there is other evidence that the wave-front bending and absence of the
Sagnac effect in the earth-centered frame is due to the clock-biasing effects of velocity
and that an ether drift velocity actually exists in the earth-centered frame. First, the
gradient of the solar gravitational effects upon clocks on the surface of the earth is such
that the clocks will speed up and slow down in precisely the correct way to retain the
appropriate up-wind and down-wind clock biases. Thus, the clocks must be biased or
else the solar gravitational effects would become apparent.


This is the reason why relativists are abandoning Einstein's version of the theory of relativity and are embracing the local-aether theory (see the first paper in this message).


Therefore, your erroneous belief in a "spinning globe" is a TOTAL FRAUD.

Title: Re: Earths Curvature and spin effect on long range ballistics.
Post by: Mysfit on October 05, 2018, 06:56:43 AM
A couple things I would like point.
#1 one main component of their job is the "kill" shot. they are assassins.
# I also think Tom misunderstood, but I'd still like to point out that using the minute measurable deflection at such short distances absolutely strengthens the argument of this effects existence..     
The manual indicates that simply hitting the target, even if just to injure, is a success. An assassin would need to guarantee a kill.
Soldiers aren’t assassins. That’s a horrible thing to call them. They are way less accurate.
Title: Re: Earths Curvature and spin effect on long range ballistics.
Post by: Curiosity File on October 05, 2018, 07:45:06 AM
A couple things I would like point.
#1 one main component of their job is the "kill" shot. they are assassins.
# I also think Tom misunderstood, but I'd still like to point out that using the minute measurable deflection at such short distances absolutely strengthens the argument of this effects existence..     
The manual indicates that simply hitting the target, even if just to injure, is a success. An assassin would need to guarantee a kill.
Soldiers aren’t assassins. That’s a horrible thing to call them. They are way less accurate.
Snipers aren't just military and even military snipers have been given orders to take out high profile non military targets. One or more snipers assassinated JKK  for example. 
Title: Re: Earths Curvature and spin effect on long range ballistics.
Post by: sandokhan on October 05, 2018, 11:39:20 AM
Here is the derivation for the full equation for g:

(http://image.ibb.co/bJJHkx/acc1.jpg)

(http://image.ibb.co/bXW3Qx/acc2.jpg)

This is the classic derivation which can be found in any number of textbooks on mechanics.

Newton kept for the public ONLY the first radial component, and eliminated all others (angular acceleration, Coriolis force).

That is why g is a variable and not a constant: once the angular acceleration/Coriolis force terms can be activated its value can be either increased or decreased.

We will see immediately that the radial component itself, -GM/r2, cannot be correct: Newton derived this equation from the fake Keplerian data.

http://citeseerx.ist.psu.edu/viewdoc/download?doi=10.1.1.424.4364&rep=rep1&type=pdf

Gravitational Induction and the Gyroscopic Force
(A hydrodynamical theory of gravity that accounts for the gyroscopic force)

(http://image.ibb.co/bJFV5x/acc3.jpg)

This equation accounts FOR ALL OF THE PHENOMENA left unexplained by Newton's single term gravitational formula: the DePalma/Kozyrev experiments, the Sagnac/Coriolis effect, the gravitational/magnetic Aharonov-Bohm effect, the Podkletnov effect, the Biefeld-Brown effect, the Nipher effect.

"We can summarize the current deficiencies in both gravitational theory and electromagnetic theory.

In gravitation, we are missing both the Coriolis/gyroscopic component and the angular acceleration component. The Coriolis/gyroscopic component is sadly missing from all contemporary accounts relating to the theory of gyroscopes."


KEPLER'S FRAUD: the faking of the astronomical data in the New Astronomia

Dr. W.H. Donahue explains how Kepler forged/falsified/faked the entries:

(https://image.ibb.co/ndKidb/kpl1.jpg)
(https://image.ibb.co/kenTdb/kpl2.jpg)
(https://image.ibb.co/js8vPG/kpl3.jpg)
(https://image.ibb.co/ch4oBw/kpl4.jpg)
(https://image.ibb.co/fFVH4G/kpl5.jpg)

A total fraud and forgery. The data from Tycho Brahe's precise observations for the circular orbit, MODIFIED to give the appearance of a nonexistent elliptical orbit: "the observational input is nil".


https://www.theflatearthsociety.org/forum/index.php?topic=30499.msg1776670#msg1776670

Kepler published his first law of planetary motion based on the data gathered by Tycho
Brahe in 1609. The law states that planets orbit the sun in ellipses with the sun at one focus.

“Almost 400 years later, William H. Donohue undertook the task of translating
Kepler’s 1609 Astronomia Nova into the English New Astronomy (Donohue 1992)
when in the course of his work he redid many of Kepler’s calculations, he was
startled to find some fundamental inconsistencies with Kepler’s reporting of these
same calculations (Donohue 1988)."

“After detailed computational arguments Donahue concluded the results
reported by Kepler . . . were not at all based on Brahe’s observational data; rather
they were fabricated on the basis of Kepler’s determination that Mars’s orbit was elliptical.



http://adsabs.harvard.edu/full/1988JHA....19..217D

Kepler's fabricated figures, by W.H. Donohue

The scholar, William H. Donahue, said the evidence of Kepler's scientific fakery is contained in an elaborate chart he presented to support his theory.

The fabricated data appear in calculated positions for the planet Mars, which Kepler used as a case study for all planetary motion. Kepler claimed the calculations gave his elliptical theory an independent check. But in fact they did nothing of the kind.

''He fudged things,'' Dr. Donahue said, adding that Kepler was never challenged by a contemporary. A pivotal presentation of data to support the elliptical theory was ''a fraud, a complete fabrication,'' Dr. Donahue wrote in his paper. ''It has nothing in common with the computations from which it was supposedly generated.''


A FRAUD AND A COMPLETE FABRICATION.

The very foundation of modern science, the most significant work in astronomy ever published, turns out to be a total fakery and forgery.


Now, here is how Newton derived his fake law of attractive gravitation from Kepler's fake data.

https://www.theflatearthsociety.org/forum/index.php?topic=30499.msg1992933#msg1992933

(https://image.ibb.co/hKmYdn/fake1.jpg)
(https://image.ibb.co/hOnH4S/fake2.jpg)

That is why the radial component, -GM/2, is totally false, it is based on the fake data introduced by Kepler.

g is a variable, and not a constant: the full acceleration equation proves it.

That is why if we can modify either the angular acceleration or the Coriolis force components, we can modify the value of g, this is the DePalma effect.

Title: Re: Earths Curvature and spin effect on long range ballistics.
Post by: Mysfit on October 05, 2018, 01:00:00 PM
The manual indicates that simply hitting the target, even if just to injure, is a success. An assassin would need to guarantee a kill.
Soldiers aren’t assassins. That’s a horrible thing to call them. They are way less accurate.
Snipers aren't just military and even military snipers have been given orders to take out high profile non military targets. One or more snipers assassinated JKK  for example.
The manual only covers US marine snipers. If you have a manual for other snipers, I would love to know if it mentions the Coriolis effect.
Soldiers have been known to be ordered to kill civilians, but this is going off topic and I don't know who JKK is and don't care unless it was done at long range and required knowledge of the Coriolis effect.
Please stop trying to make this about people, the manual is clear and the video is undisputed evidence. We are doing well.
Title: Re: Earths Curvature and spin effect on long range ballistics.
Post by: Curiosity File on October 05, 2018, 03:00:33 PM
The manual indicates that simply hitting the target, even if just to injure, is a success. An assassin would need to guarantee a kill.
Soldiers aren’t assassins. That’s a horrible thing to call them. They are way less accurate.
Snipers aren't just military and even military snipers have been given orders to take out high profile non military targets. One or more snipers assassinated JKK  for example.
The manual only covers US marine snipers. If you have a manual for other snipers, I would love to know if it mentions the Coriolis effect.
Soldiers have been known to be ordered to kill civilians, but this is going off topic and I don't know who JKK is and don't care unless it was done at long range and required knowledge of the Coriolis effect.
Please stop trying to make this about people, the manual is clear and the video is undisputed evidence. We are doing well.

JFK typo corrected.
But I agree this is completely off topic.
Title: Re: Earths Curvature and spin effect on long range ballistics.
Post by: stack on October 05, 2018, 04:54:47 PM
We know that there are a lot of people who think it exists and write about it. However, it would be a much stronger argument to show that there have been artillery experiments on reality to directly demonstrate the matter, rather than words about theory or supposition of what slight adjustments should be made.

As part of artillery sighting enhancements, the Army tested a new ranging system (JETS). It uses, in part, a Precision Azimuth Vertical Angle Module (PAVAM).

"Soldiers with the 2nd Battalion, 319th Field Artillery Regiment spent four days testing the Joint Effects Targeting System.
The system is a man-portable, handheld, target observation, location and designation system built for day or night use in all weather conditions, according to an Army Operational Test Command release.
The system includes a Handheld Target Location Module, a Laser Marker Module and a Precision Azimuth Vertical Angle Module, which are mounted on a tripod."

https://www.armytimes.com/news/your-army/2017/09/20/artillery-soldiers-airdrop-test-new-targeting-system/

"Thanks to its robust algorithms and high performance HRG CrystalTM gyroscopes, PAVAM finds true north very quickly by sensing the earth's rotation and determining the azimuth to north...We are happy to provide PAVAM to the Soldier in partnership with Leonardo DRS and the Army."

https://www.safran-electronics-defense.com/media/more-100-precision-azimuth-vertical-angle-module-pavam-delivered-safran-optics-1-2017-20180611

"The PAVAM uses the spin of the earth to help the JETS provide accurate targeting information.”

https://www.army.mil/article/175956/army_awards_contract_for_jets_handheld_targeting_system

More on JETS Testing:

From the U.S. Indo-Pacific Command:

"After spending a month with the targeting systems, most Soldiers were ready for the system to be fielded.”

http://www.pacom.mil/Media/News/News-Article-View/Article/1468797/field-artillery-soldiers-test-new-precision-targeting-system-to-find-friend-or/

"The Army said at the end of 2017 that it expected to wrap up JETS testing in early 2018 and have the system in the hands of every forward-observation team by the middle of the year.”

https://www.businessinsider.com/us-soldiers-testing-joint-effects-targeting-system-2018-3
Title: Re: Earths Curvature and spin effect on long range ballistics.
Post by: Tom Bishop on October 05, 2018, 09:06:26 PM
The JETS is this computerized spotting device on a tripod:

(https://www.armytimes.com/resizer/zjD7sB6GvWmbRIUYKEubMSuV9os=/1200x0/filters:quality(100)/arc-anglerfish-arc2-prod-mco.s3.amazonaws.com/public/M73GMTSB4ZDZNIEYI7WUFRA224.jpg)

I have do no doubt at all that the calculations that such a device can attempt to account for the supposed "Coriolis Effect."

The forward observer transmits the target information to the artilleryman, and then the Artilleryman fires on it and proceeds to miss the target! (https://wiki.tfes.org/The_Coriolis_Effect#Artillery_Ballistics_Not_Accurate) As we have documented, military artillery is not accurate. JETS is merely a computerized version of the Artillery Coriolis Table that we saw.

If the location data is transmitted to a weapon with guided munitions, such a guided missile, then the missile will seek the target coordinates regardless of wind, weather, trajectory, imperfections in flight, or the "Coriolis Effect".
Title: Re: Earths Curvature and spin effect on long range ballistics.
Post by: stack on October 05, 2018, 09:22:27 PM
The JETS is this computerized spotting device on a tripod:

(https://www.armytimes.com/resizer/zjD7sB6GvWmbRIUYKEubMSuV9os=/1200x0/filters:quality(100)/arc-anglerfish-arc2-prod-mco.s3.amazonaws.com/public/M73GMTSB4ZDZNIEYI7WUFRA224.jpg)

I have do doubt at all that the calculations that such a device can make attempt to account for the supposed "Coriolis Effect."

Can't say that it does exactly, but your 'doubt' is based on nothing and really means nothing.   It does, however, reference the fact that the earth rotates:

"The PAVAM uses the spin of the earth to help the JETS provide accurate targeting information.”

https://www.army.mil/article/175956/army_awards_contract_for_jets_handheld_targeting_system

The forward observer transmits the target information to the artilleryman, and then the Artilleryman fires on it and proceeds to miss the target! (https://wiki.tfes.org/The_Coriolis_Effect#Artillery_Ballistics_Not_Accurate) As we have documented, military artillery is not accurate. JETS is merely a computerized version of the Artillery Coriolis Table that we saw.

Now you're saying that JETS does does factor in the coriolis effect? I'm confused, up above you stated that you doubted it did.

If the location data is transmitted to a weapon with guided munitions, such a guided missile, then the missile will seek the target coordinates regardless of wind, weather, trajectory, imperfections in flight, or the "Coriolis Effect".

I thought we were talking about artillery, not guided missiles?



Title: Re: Earths Curvature and spin effect on long range ballistics.
Post by: Tom Bishop on October 05, 2018, 09:31:20 PM
Quote
Can't say that it does exactly, but your 'doubt' is based on nothing and really means nothing.

There was a slight typo on my last message. I said that I have no doubt that is does what it claims to do and can make the calculations that it claims to perform.

"The PAVAM uses the spin of the earth to help the JETS provide accurate targeting information.”

This U.S. Army Artillery Coriolis Table (https://wiki.tfes.org/The_Coriolis_Effect#U.S._Army_Artillery_Coriolis_Table_Example) also attempts to provide target information to account for the supposed rotation of the earth. What is your point? Is your point that someone was able to put it onto a computer and sell it to the government for lots of money?
Title: Re: Earths Curvature and spin effect on long range ballistics.
Post by: stack on October 05, 2018, 09:37:07 PM
Quote
Can't say that it does exactly, but your 'doubt' is based on nothing and really means nothing.

I said that I have no doubt that is does what it claims to do and can make the calculations that it claims to perform.

"The PAVAM uses the spin of the earth to help the JETS provide accurate targeting information.”

This U.S. Army Artillery Coriolis Table (https://wiki.tfes.org/The_Coriolis_Effect#U.S._Army_Artillery_Coriolis_Table_Example) also attempts to provide target information to account for the supposed rotation of the earth. What is your point? Is your point that someone was able to put it onto a computer and sell it to the government for lots of money?

You wrote "I have do doubt..." which I inferred that you meant 'to' doubt. My apologies.

My point is that you claim there is no demonstration or experimentation as to factoring in a rotating earth when it comes to long range artillery. This device does factor in a rotating earth and has been tested buy the military. And is being deployed to troops. Simple as that.